You are on page 1of 59

7/7/2015

about:blank

2015 Bull Mock CAT - 19


DIRECTIONS for the question: Analyse the graph/s given below and answer the question that follows.
Question No. : 1
The total production of Wheat, Sugarcane, Rice, Soya Bean and Maize in 1980 was 324 million tonnes and in 2010 was 1024
million tonnes. The diagrams below show the percent share of each of these five crops in total production in the respective years.
The outermost square in each diagram has an area of 100 and all figures drawn are regular.

If the five crops were ranked according to their respective shares in the total production, for how many crops has the rank
changed from 1980 to 2010?
A)5

B)3

C)4

D)2

Explanation:-

The ranks, from 1 to 5 in 1980 were Sugarcane, Rice, Wheat, Soya Bean and Maize respectively and in 2010 were Sugarcane,
Wheat, Rice, Soya Bean and Maize respectively.
Thus, the ranks for two crops changed from 1980 to 2010.

http://www.hitbullseye.com/frmTestView.aspx

1/59

7/7/2015

about:blank

DIRECTIONS for the question: Analyse the graph/s given below and answer the question that follows.
Question No. : 2
The total production of Wheat, Sugarcane, Rice, Soya Bean and Maize in 1980 was 324 million tonnes and in 2010 was 1024
million tonnes. The diagrams below show the percent share of each of these five crops in total production in the respective years.
The outermost square in each diagram has an area of 100 and all figures drawn are regular.

For how many crops was the production in 2010 greater than that in 1980?
A)5

B)3

C)4

D)1

Explanation:-

One option is to actually calculate the values for production of the five crops across both the years and then compare.
From the given information, we know that the total production in 2010 was more than thrice that in 1980.
So, from the point of view of approximation, if we multiply all the percentages in 2010 by 3, we find that all the values will be
greater than the corresponding percentages in 1980.
Thus, the production for all five crops was greater in 2010 than in 1980.

http://www.hitbullseye.com/frmTestView.aspx

2/59

7/7/2015

about:blank

DIRECTIONS for the question: Analyse the graph/s given below and answer the question that follows.
Question No. : 3
The total production of Wheat, Sugarcane, Rice, Soya Bean and Maize in 1980 was 324 million tonnes and in 2010 was 1024
million tonnes. The diagrams below show the percent share of each of these five crops in total production in the respective years.
The outermost square in each diagram has an area of 100 and all figures drawn are regular.

In 2010, the production of Sugarcane was approximately what percent of that of Wheat?
A)240%

B)450%

C)150%

D)320%

Explanation:-

http://www.hitbullseye.com/frmTestView.aspx

3/59

7/7/2015

about:blank

In 2010, the production of Wheat and Sugarcane was 19.625% and 50% of the total production.
The required percentage is 30/19.625 30/20 = 150%.
The best answer is option 3.
DIRECTIONS for the question: Analyse the graph/s given below and answer the question that follows.
Question No. : 4
The total production of Wheat, Sugarcane, Rice, Soya Bean and Maize in 1980 was 324 million tonnes and in 2010 was 1024
million tonnes. The diagrams below show the percent share of each of these five crops in total production in the respective years.
The outermost square in each diagram has an area of 100 and all figures drawn are regular.

For how many crops was the production in 2010 more than twice the production in 1980?
A)3

B)5

C)1

D)2

Explanation:-

http://www.hitbullseye.com/frmTestView.aspx

4/59

7/7/2015

about:blank

One option is to actually calculate the values for production of the five crops across both the years and then compare.
From the given information, we know that the total production in 2010 was more than thrice that in 1980.
So, from the point of view of approximation, if we multiply all the percentages in 2010 by 3, we find that the values will be
greater than twice the corresponding percentages in 1980 for Wheat, Soya Bean and Sugarcane.
Thus, the production for three crops in 2010 was more than twice the production of those crops in 1980.

Question No. : 5
DIRECTIONS for the question: Analyse the graph/s given below and answer the question that follows.

The table below shows the Revenue and Net Profit for different companies in 2006 and 2007 while the bar graph shows the
amount of taxes paid by these companies over the two years. Gross profit is calculated as Revenue less Operating Costs and 30%
of Gross Profit is deducted as Taxes to arrive at Net Profit. All values are in Rs. Crore.

2006

2007

Revenue

Net Profit

Revenue

Net Profit

Ambuja Cements

6274

2196

5704

1597

ACC

5984

1466

7189

2768

Birla Corporation

2049

717

1996

419

Chettinad Cement

1110

311

1322

370

Dalmia Cement

1692

651

1973

483

India Cements

3839

672

3554

498

Madras Cements

2905

1322

2335

736

Prism Cement

1021

250

722

152

Shree Cements

2440

512

3097

867

UltraTech Cement

6286

1760

7160

3007

Total

33600

9858

35052

10897

http://www.hitbullseye.com/frmTestView.aspx

5/59

7/7/2015

about:blank

For which of the following companies was the total Operating Cost for the two years greater than 70% of the total revenue for the
two years?
A)Birla Corporation

B)India Cement

C)UltraTech Cement

D)Chettinad Cement

Explanation:-
Since Net Profit = 70% of Gross Profit, Gross Profit can be calculated as Net Profit (100/70). Operating cost can be calculated as
Revenue Gross Profit.
Bir. Corp.: GP = (717 + 419) (100/70) = 1622. Op.Cost = (2049 + 1996) 1622 = 2423. The required percentage is 2423/4045
= 60%.
Ind. Cem.: GP = (672 + 498) (100/70) = 1671. Op.Cost = (3839 + 3554) 1671 = 5722. The required percentage is 5722/7393
= 77%.
UltTech: GP = (1760 + 3007) (100/70) = 6810. Op.Cost = (6286 + 7160) 6810 = 6636. The required percentage is
6636/13446 = 49%.
Chet. Cem.: GP = (311 + 370) (100/70) = 972. Op.Cost = (1110 + 1322) 972 = 1460. The required percentage is 1460/2432 =
60%.
It is more than 70% for india cement. Hence second option.

Question No. : 6
DIRECTIONS for the question: Analyse the graph/s given below and answer the question that follows.

The table below shows the Revenue and Net Profit for different companies in 2006 and 2007 while the bar graph shows the
amount of taxes paid by these companies over the two years. Gross profit is calculated as Revenue less Operating Costs and 30%
of Gross Profit is deducted as Taxes to arrive at Net Profit. All values are in Rs. Crore.

2006

2007

Revenue

Net Profit

Revenue

Net Profit

Ambuja Cements

6274

2196

5704

1597

ACC

5984

1466

7189

2768

Birla Corporation

2049

717

1996

419

Chettinad Cement

1110

311

1322

370

Dalmia Cement

1692

651

1973

483

India Cements

3839

672

3554

498

Madras Cements

2905

1322

2335

736

Prism Cement

1021

250

722

152

http://www.hitbullseye.com/frmTestView.aspx

6/59

7/7/2015

about:blank

Shree Cements

2440

512

3097

867

UltraTech Cement

6286

1760

7160

3007

Total

33600

9858

35052

10897

For all the companies together, the total Gross Profit over the two years was approximately what percent of the total Revenue over
the two years?
A)53%

B)76%

C)30%

D)43%

Explanation:-
Total net profit = (9858 + 10897) = 20,755
Total Gross Profit = 20755 (100/70) = 29650.

Total revenue over the two years = (33600 + 35052) = 68,652


The required percentage is (29650 / 68652 ) * 100 = 43% (approx)

Question No. : 7
DIRECTIONS for the question: Analyse the graph/s given below and answer the question that follows.

The table below shows the Revenue and Net Profit for different companies in 2006 and 2007 while the bar graph shows the
amount of taxes paid by these companies over the two years. Gross profit is calculated as Revenue less Operating Costs and 30%
of Gross Profit is deducted as Taxes to arrive at Net Profit. All values are in Rs. Crore.

2006

2007

Revenue

Net Profit

Revenue

Net Profit

Ambuja Cements

6274

2196

5704

1597

ACC

5984

1466

7189

2768

Birla Corporation

2049

717

1996

419

Chettinad Cement

1110

311

1322

370

Dalmia Cement

1692

651

1973

483

India Cements

3839

672

3554

498

Madras Cements

2905

1322

2335

736

Prism Cement

1021

250

722

152

Shree Cements

2440

512

3097

867

UltraTech Cement

6286

1760

7160

3007

Total

33600

9858

35052

10897

http://www.hitbullseye.com/frmTestView.aspx

7/59

7/7/2015

about:blank

For how many companies did the Revenue increase from 2006 to 2007 but the Operating Cost decreased from 2006 to 2007?
A)5

B)0

C)2

D)3

Explanation:-
The revenues of ACC, Chettinad Cement, Dalmia Cement, Shree Cement and UltraTech Cement increased over the 2 years.
We can calculate the Op. Cost as Revenue (100 Net Profit)/70.
ACC: Op. Cost06 = 5984 (146600/70) = 3890. Op. Cost07 = 7189 (276800/70) = 3235.
Chet. Cem: Op. Cost06 = 1110 (31100/70) = 665. Op. Cost07 = 1322 (37000/70) = 793.
Dal. Cem: Op. Cost06 = 1692 (65100/70) = 762. Op. Cost07 = 1973 (48300/70) = 1283.
Shree Cem: Op. Cost06 = 2440 (51200/70) = 1708. Op. Cost07 = 3097 (86700/70) = 1858.
UltTech: Op. Cost06 = 6286 (176000/70) = 3771. Op. Cost07 = 7160 (300700/70) = 2864.
Thus, only 2 companies, ACC and UltraTech satisfy the given criteria.

Question No. : 8
DIRECTIONS for the question: Analyse the graph/s given below and answer the question that follows.

The table below shows the Revenue and Net Profit for different companies in 2006 and 2007 while the bar graph shows the
amount of taxes paid by these companies over the two years. Gross profit is calculated as Revenue less Operating Costs and 30%
of Gross Profit is deducted as Taxes to arrive at Net Profit. All values are in Rs. Crore.

2006

2007

Revenue

Net Profit

Revenue

Net Profit

Ambuja Cements

6274

2196

5704

1597

ACC

5984

1466

7189

2768

Birla Corporation

2049

717

1996

419

Chettinad Cement

1110

311

1322

370

Dalmia Cement

1692

651

1973

483

India Cements

3839

672

3554

498

Madras Cements

2905

1322

2335

736

Prism Cement

1021

250

722

152

Shree Cements

2440

512

3097

867

UltraTech Cement

6286

1760

7160

3007

Total

33600

9858

35052

10897

http://www.hitbullseye.com/frmTestView.aspx

8/59

7/7/2015

about:blank

For which of the following companies did the amount of Taxes paid decrease from 2006 to 2007, but the Revenue increased from
2006 to 2007?
A)Ambuja Cement

B)Dalmia Cement

C)India Cement

D)Shree Cement

Explanation:-
From the table we see that Revenues increased for Dalmia Cement and Shree Cement.

Dalmia.Cement.: Taxes06 = 279. Taxes07 = 207.


Shree. Cem.: Taxes06 = 220. Taxes07 = 372.

Thus, for Dalmia Cement the taxes decrease and at the same time revenues increase.

DIRECTIONS for the question: Solve the following question and mark the best possible option.
Question No. : 9
If A1, A2, A3,. A100 are all natural numbers, not necessarily distinct, such that A12=A22+A32+A42++A1002, then at least how
many of them are divisible by 3?
A)2

B)3

C)1

D)0

Explanation:-
A12=A22+.+A1002

If x is divisible by 3, x2 is also divisible by 3. If x is not divisible by 3. x2 leaves a remainder of 1, when divisible by 3.


If none of the numbers in the RHS are divisible by 3, each would leave a remainder of 1 and the RHS would leave the same
remainder as 99. i.e. 0 and thus A1 would have to be divisible by 3.
At least one number has to be divisible by 3.

DIRECTIONS for the question: The following question contains two statements marked I and II. Mark the correct answer.
Question No. : 10
The set P is defined as the set of positive integers such that if the integer x is in set P, then x2 and x3 are also in set P. A rootinteger
is defined as an integer y in set P such that y is the only integer in set P which is not the squareor cube of any otherinteger in set
P. Is 8 in set P?

I. 64 is in set P and 64 is not the root integer.


II. 4 is in set P and 4 is not the root integer.

A) If the question can be answered with the help of any one statement alone, but not with the help of the other statement

http://www.hitbullseye.com/frmTestView.aspx

9/59

7/7/2015

about:blank

A) If the question can be answered with the help of any one statement alone, but not with the help of the other statement

alone
B)If the question can be answered with the help of both statements together
C)If the question can be answered with the help of either statement alone
D)If the question cannot be answered even with the help of both statements together
Explanation:-
From statement I, since 64 is not the root integer, it must be the square or cube of another integer. If 64 is taken as the square of
an integer, i.e., 64 = 82, then 8 is in P. On the other hand, if 64 is taken as the cube of an integer, i.e., 64 = 43, then 4 is in P. If 2 is in
set P, then 4 and 8 must be in set P. However, there is no information about whether 2 is in set P. SO we cannot say whether 8 is in
set P. Thus statement I alone is not sufficient to answer the question. From statement II, since 4 = 22 is in P, 2 is also in P that means
23 = 8 is also in P.Sostatement II alone is sufficient to answer the question.
DIRECTIONS for the question: Solve the following question and mark the best possible option.
Question No. : 11
What is the radius (in cm) of the biggest possible circle that can be inscribed in a sector of radius 10 cm and a central angle of
1200?
A)

B)

C)

D)5

Explanation:-

In the 1200 sector of radius 10 cm the biggest possible circle is to be drawn. The biggest circle when drawn will touch the two radii
and the arc of the sector at its midpoint C. Now draw a tangent at the midpoint of the arc and extend it in either direction and
also extend two radii so as to meet the tangent at A and B. The triangle OAB is similar to the isosceles triangle OPQ. Since the
altitude (which is also a median) of the triangle OAB is twice that of OPQ, all the dimensions (lengths) of the triangle OAB will be
twice that of OPQ.
Now the radius of the circle required is the inradius of the OAB which is twice the inradius of OPQ.

Alternative Solution
After drawing a basic diagram, it can be observed that the diameter of the circle is slightly less than 10 cm. Hence, the radius must
be close to 5 cm. but less than 5 cm. An approximate calculation of the answer choices shows that only choice 3 is possible. Choice
3

DIRECTIONS for the question: Answer the question independently of any other question.
Question No. : 12

A)66

B)38

C)40

D)48

Explanation:-

http://www.hitbullseye.com/frmTestView.aspx

10/59

7/7/2015

about:blank

DIRECTIONS for the question: Solve the following question and mark the best possible option.
Question No. : 13

A)2/3

B)1/3

C)3/4

D)Cannot be determined

Explanation:-
Given 2a2+17b2+8c2-6ab-20bc=0

Now, the above expression, on the L.H.S. resembles the sum of two expressions of the form (ma+nb)2 and (pb+qc)2, where m,n,p
and q are some constants. By a little trial and error , it can be seen that by multiplying the entire equation with 2, it is possible to
easily arrive at a set of m, n, p and q

(Because the only way equation (2) can be true is if each term on the L.H.S individually , equals zero)

DIRECTIONS for the question: Solve the following question and mark the best possible option.
Question No. : 14
Two hill stations A and B are separated by a distance of 600 km and the distance for which the road from A to B runs uphill is one
third of the distance for which it runs downhill. Further, the distance for which the road runs on level surface is 240 km. For a
round trip from A to B and back, the vehicle used 111 litres of petrol , Further it is known that
When travelling at a certain speed the consumption of petrol in litres/ km, by a vehicle increases by 1/4th during uphill journey
and decreases by half during downhill journey, both when compared to a normal journey, i.e. on level surface and at the same
speed.

What is the mileage (in km/litre) given by the vehicle for the downhill journey?

A)10

B)12.5

C)20

D)25

Explanation:-

http://www.hitbullseye.com/frmTestView.aspx

11/59

7/7/2015

about:blank

,
Alternative Solution
As the journey is a round trip, the total distance traveled uphill is equal to that covered downhill=600-240=360km.
Let x be the fuel consumption (in litres/km) for the round trip journey

DIRECTIONS for the question: Solve the following question and mark the best possible option.
Question No. : 15
Two hill stations A and B are separated by a distance of 600 km and the distance for which the road from A to B runs uphill is one
third of the distance for which it runs downhill. Further, the distance for which the road runs on level surface is 240 km. For a
round trip from A to B and back, the vehicle used 111 litres of petrol , Further it is known that
When travelling at a certain speed the consumption of petrol in litres/ km, by a vehicle increases by 1/4th during uphill journey
and decreases by half during downhill journey, both when compared to a normal journey, i.e. on level surface and at the same
speed.

The quantity (in litres) of petrol consumed for the return journey from B to A is

A)55.55

B)62.25

C)74

D)None of these

Explanation:-

Hence the fuel consumed for the return journey=622.5x=62.25 litres.

DIRECTIONS for the question: Solve the following question and mark the best possible option.
Question No. : 16
If a, b, c are positive real numbers such that a + [b] + {c} = 3.8, [a] + {b} + c = 3.2 and {a} + b + [c] = 2.2, what is the value of [a2] +
[b2] + [c2]? Where, [k] denotes the greatest integer less than or equal to k, and {k} denotes the fractional part of k, e.g. {2.35} = 0.35
A)6

B)7

C)8

D)9

http://www.hitbullseye.com/frmTestView.aspx

12/59

7/7/2015

about:blank

Explanation:-
We have,
a + [b] + {c} = 3.8 (1)
[a] + {b} + c = 3.2 (2)
{a} + b + [c] = 2.2 (3)
Adding the above three equations we get,
2(a + b + c) = 9.2 (Since, a, b and c are positive real numbers)
Or, a + b + c = 4.6 (4)
Subtracting equation (1) from equation (4) we get,
{b} + [c] = 0.8
{b} = 0.8 and [c] = 0,
Subtracting equation (2) from equation (4) we get,
{a} + [b] = 1.4
{a} = 0.4 and [b] = 1,
Subtracting equation (3) from equation (4) we get,
[a] + {c} = 2.4
[a] = 2 and {c} = 0.4
So, a = 2.4, b = 1.8 and c = 0.4
Hence, [a2] + [b2] + [c2] = 5 + 3 + 0 = 8

DIRECTIONS for the question: Answer the question independently of any other question.
Question No. : 17

The arithmetic mean of 3 numbers is x. It falls by 2 if one of the numbers is excluded. It is also known that this number is the
largest of the 3. Given that all the numbers are positive integers, what is the maximum value that the middle number can take?
A)2x 5

B)2x 4

C)x + 4

D)Insufficient data

Explanation:-

DIRECTIONS for the question: Answer the question independently of any other question.

Question No. : 18

A group of ladies is gathered for a cake making festival. There is a huge pile of dough in front of them. The first lady takes the required
quantity of dough by dividing the initial pile of dough into m or n smaller piles. The next lady, in her turn, divides one of the existing piles only
into m or n smaller piles and so on. For which of the following values of (m, n) is it NOT possible to get 14 piles of dough?

A)(2, 5)

B)(3, 6)

C)(4, 6)

D)(3, 9)

Explanation:-
Suppose (m, n) = (2, 5). Each time a pile is divided into 2 or 5, the number of piles increases by 1 or 4 respectively. So it is possible to get
14 piles by making 13 divisions by 2 each or 3 divisions by 5 and one division by 2. Suppose (m, n) = (3, 6). Each time a pile is divided into
3 or 6, the number of piles increases by 2 or 5 respectively. So it is possible to get 14 piles by making 4 divisions by 3 and one division by
6. Suppose (m, n) = (4, 6). Each time a pile is divided into 4 or 6, the number of piles increases by 3 or 5 respectively. So it is possible to
get 14 piles by making one division by 4 and 2 divisions by 6. Suppose (m, n) = (3, 9). Each time a pile is divided into 3 or 9, the number of
piles increases by 2 or 8 respectively. So the number of piles will only become 3, 5, 9, etc. Since 14 is an even number, it is not possible to
make 14 piles if (m, n) = (3, 9).

DIRECTIONS for the question: Answer the question independently of any other question.

Question No. : 19

Amardeep Singh, Banreet Kaur and Dhirender Singh ordered a pizza and cut it into 5 equal parts. They distributed the 5 pieces among
themselves such that each of them got at least one piece of pizza and none of them got pieces that were adjacent. In how many ways
could they share the pizza?

A)30

B)24

C)42

D)18

http://www.hitbullseye.com/frmTestView.aspx

13/59

7/7/2015

about:blank

Explanation:-

DIRECTIONS for the question: Answer the question independently of any other question.
Question No. : 20

A)-3/2

B)-17/8

C)-1

D)Insufficient Data

Explanation:-

DIRECTIONS for the question: Answer the question independently of any other question.

Question No. : 21

The distance between towns A and B is 555 km. An aeroplane took off from town A towards town B at a constant speed of 105 kmph. After
some time, the tailwind picked up and the remaining part of the journey was flown at a constant speed of 115 kmph. If the entire journey
took 5 hours, for how long did the aeroplane fly at 105 kmph?

A)3 hours

B)2 hours

C)3 hours

D)2 hours

Explanation:-

DIRECTIONS for the question: Solve the following question and mark the best possible option.

http://www.hitbullseye.com/frmTestView.aspx

14/59

7/7/2015

about:blank

Question No. : 22
p, q, r and s are positive real numbers such that p + q + r + s = x, where x is a constant. Find the maximum value of (x p)(x q)
(x r)(x s)?
A)(x4 pqrs)/64

B)81x4/256

C)27x3/512

D)9x4/4

Explanation:-

DIRECTIONS for the question: Answer the question independently of any other question.
Question No. : 23

The first term of a sequence is 1 and the second term is 5. From the third term onwards, each term is the average of all the
preceding terms. What is the difference between the 30th and 25th terms of this sequence?
A)3

B)0

C)6

D)12

Explanation:-
The 3rd term is (1 + 5)/2 = 6/2 = 3, the 4th term is (6 + 3)/3 = 9/3 = 3, the 5th term is (9 + 3)/4 = 12/4 = 3.
Since each of the remaining terms is 3, the 25th and 30th terms are also 3.
The difference is 0.

DIRECTIONS for the question: Solve the following question and mark the best possible option.
Question No. : 24
A shopkeeper bought an article for Rs.1000 and marked its price as Rs.2160. He gave three successive discounts of a%, b% and
c%, where a + b + c = 50. If he made a profit of x% finally, how many of the following ranges contain values which are NOT
possible values of x?

A)1

B)2

C)0

D)3

Explanation:-
The 3 successive discounts are a%, b% and c%, where a + b + c = 50%
The maximum discount(x%), would occur when a=50%, b = c = 0%
In this case, selling price=1080. Profit% = 8%

There two cases are represented below.


Minimum Selling Price 1080

Maximum Selling Price 1250

Minimum Profit percentage 8% Maximum Profit Percentage 25%

http://www.hitbullseye.com/frmTestView.aspx

15/59

7/7/2015

about:blank

DIRECTIONS for the question: Answer the question independently of any other question.

Question No. : 25

A goldsmith has n gold coins of weights 10 gm, 20 gm, 30 gm, , 10n gm (n = 1, 2, 3, ..). While locking up his store at night, he locks up
the gold coins in three different safes such that the first safe holds the gold coins weighing 10 gm, 40 gm, 70 gm and so on, the second
safe holds the gold coins weighing 20 gm, 50 gm, 80 gm and so on and the third safe holds the rest of the gold coins. The goldsmith
realises that the average weights of the gold coins in exactly two of the three safes were the same as the weight of one of the gold coins in
the respective safes and the average weight of the gold coins in the other safe was different from any of the gold coins in that safe. Which
of the following could be the total number of gold coins?

A)113

B)90

C)82

D)99

Explanation:-

DIRECTIONS for the question: Answer the question independently of any other question.

Question No. : 26

What is the remainder when 3040 is divided by 17?

A)13

B)1

C)4

D)16

Explanation:-

DIRECTIONS for the question: Answer the question independently of any other question.

Question No. : 27

10-digit positive integers are formed by using the digits 2 and 3 so that the integer has at least one 2 and one 3 and no two 3s come
together. How many of these integers are divisible by 3?

A)101

B)35

C)45

D)72

Explanation:-

DIRECTIONS for the question: Answer the question independently of any other question.

Question No. : 28

http://www.hitbullseye.com/frmTestView.aspx

16/59

7/7/2015

about:blank

The Rajdhani Express and the Shatabdi Express are travelling towards each other at speeds of 54 kmph and 72 kmph respectively. Ram is
sitting at the tail end of the Rajdhani Express which is 340 m long and Shyam is sitting at the front end of the Shatabdi express which is 500
m long. When the two trains start to cross each other, Ram starts running towards the front end of the Rajdhani Express at a speed of 2
m/s and Shyam starts running towards the tail end of the Shatabdi Express at a speed of 3 m/s. After how much time will Ram and Shyam
be at the same point?

A)12 seconds

B)10 seconds

C)17 seconds

D)20 seconds

Explanation:-
The Rajdhani Express is travelling at 54 kmph = 15 m/s. Since Ram is running in the same direction as the train, his effective speed is 15
+ 2 = 17 m/s. The Shatabdi Express is travelling at 72 kmph = 20 m/s. Since Shyam is running in the opposite direction as the train, his
effective speed is 3 20 = 17 m/s. This means that the trains speed is much more than Shyams. The two of them are 340 m apart and
their relative speed is 17 (17) = 34 m/s. Thus the time required is 340/34 = 10 seconds.

DIRECTIONS for the question: Answer the question independently of any other question.
Question No. : 29
Rahul and Vijay are standing on adjacent sides of a rectangular field at points P and Q respectively. The points P and Q divide the
length and breadth in the ratio 3 : 1 and 2 : 1 respectively. Rahul is standing at point P on the length of the field so that he is at the
shortest possible distance from the common vertex and Vijay is at point Q on the breadth so that he is at the largest possible
distance from the common vertex. If PQ is 100 m, and the sides of the field are integers, what is the area of the field?
A)27,555 m2

B)25,980 m2

C)28,800 m2

D)Insufficient data

Explanation:-
Triangle formed by points P, Q and common vertex is a right triangle with hypotenuse 100. Since the sides of the field are integers,
2y and x take values 60 and 80 respectively, or 80 and 60 respectively. Accordingly, the length and breadth are 320 and 90
respectively or 240 and 120 respectively. In either case, the area is 28800 m2.

DIRECTIONS for the question: Answer the question independently of any other question.

Question No. : 30

A)9/8

B)10/9

C)1

D)0

Explanation:-

DIRECTIONS for the question: Answer the following question as per the best of your judgment.
Question No. : 31
A fruit seller sells an apple, an orange, a mango and a peach at loss of 10%, loss of 40%, profit of 20% and profit of 50%
respectively. Let the loss made on an apple and an orange together be as much as in amount as the profit made on a mango and a
peach together. Suppose you have the money to buy one apple and four oranges. But instead you buy five peaches. How many
mangoes can you buy with the rest of the money?
A)1

B)2

C)4

D)Cant say

http://www.hitbullseye.com/frmTestView.aspx

17/59

7/7/2015

about:blank

Explanation:-
Let the cost prices of an apple, an orange, a mango and a peach be w, x, y and z respectively. Then 0.1 w + 0.4 x = 0.2 y + 0.5 z, w
+ 4x = 2y + 5z. Now the amount we have = 0.9w + 0.6 4x. Let the required number of mangoes we can buy be n, then 0.9w +
2.4x = 1.2yn + 1.5z 5, 0.9 w + 2.4 x = 1.2 y n + 1.5 (w + 4x 2y)=> (3 1.2n) y = 0.6w + 3.6x. Therefore, n cannot be
determined.

DIRECTIONS for the question: Answer the following question as per the best of your judgment.
Question No. : 32
In how many ways can one choose 6 cards from a normal deck of cards so as to have all suits present?
A)134 48 47

B)134 27 47

C)134

D)None of these

Explanation:-
52 cards in a deck -13 cards per suit
First card - let us say from suit hearts = 13C1 =13

Second card - let us say from suit diamonds = 13C1 =13


Third card - let us say from suit spade = 13C1 = 13
Fourth card - let us say from suit clubs = 13C1=13
Remaining cards in the deck= 52 4 = 48.
Fifth card - any card in the deck = 48C1
Sixth card - any card in the deck = 47C1

Total number of ways = 13 13 13 13 48 47 = 134 48 47

DIRECTIONS for the question: Answer the following question as per the best of your judgment.
Question No. : 33
Find the number of two-digit numbers [where neither digit is zero] whose product of the digits is a perfect square?
A)16

B)17

C)18

D)None of these

Explanation:-
Numbers are 11, 22, 33_______99, 14, 19, 28, 41, 49, 82, 94, 91.... Hence there are 17 such numbers. Second option is the answer.

DIRECTIONS for the question: Answer the following question as per the best of your judgment.
Question No. : 34
Ashok has some 5 rupee and some 2 rupee coins. He finds that he cannot choose a set of coins out of his collection whose total
value is Rs. 21. However, he can choose a set of coins whose total value is Rs. 65. Which of these statements could be true?
A)Ashok has only one 5 rupee coin B)Ashok has only five 2 rupee coins
D)Ashok has only seven 5 rupee coins

C)Ashok has only one 2 rupee coin

Explanation:-
Since he cannot choose a set of coins out of his collection whose total value is Rs. 21. Therefore number of 2 Rupee coins must be
less than or equal to two because if 3 or more 2 Rupee coins are available then we can make a set of coins whose value is 21 by
using 3 two Rupee coins and 3 five Rupee coins. Hence 3rd option is the only possible answer.

DIRECTIONS for the question: Answer the following question as per the best of your judgment.
Question No. : 35
Rahim plans to drive from city A to station C, at the speed of 70 km per hour, to catch a train arriving there from B. He must reach
C at least 15 minutes before the arrival of the train. The train leaves B, located 500 km south of A, at 8:00 am and travels at a speed
http://www.hitbullseye.com/frmTestView.aspx

18/59

7/7/2015

about:blank

of 50 km per hour. It is known that C is located between west and northwest of B, with BC at 60 to AB. Also, C is located between
south and southwest of A with AC at 30 to AB. The latest time by which Rahim must leave A and still catch the train is closest to
A)5:15 am

B)6:30 am

C)6:45 am

D)7:00 am

Explanation:-
The angles of the triangle formed by A, B and C tell us that ABC is a right-angle triangle, with right-angle at vertex C, 30 at vertex
A and 60 at vertex B. Since AB = 500 km, in 30-60-90 triangle ABC, we get, AC= 250*3 km and BC = 250 km. The train
travels at 50 km/hr. It will travel from B to C (i.e. 250 km) in 5 hours. Since it leaves at 8:00 a.m., it will reach C at 1:00 p.m. Now,
Rahim must be at C latest by 12:45 p.m. (15 minutes before the train). Travelling at 70 km/hr, he will take approximately 6.2
hours or 6 hours 12 min to travel from A to C. Now 6 hours 12 minutes before 12:45 pm the time will be close to 6:30am. If he
leaves by 6:45 a.m., he will not make it to point C, 15 minutes before the train arrives. Hence second option is the answer.

DIRECTIONS for the question: Solve the following question and mark the best possible option.
Question No. : 36
The daily wages of Kalu and Bawa and also of Bawa and Lalit are in the ratio 2 : 3 each. A third of Lalits wages exceeds half of
Kalus wages by Rs. 80. Each of them spends the same amount of money and their savings are in the ratio 1 : 9 : 21. What is their
combined expenditure?
A)Rs. 300

B)Rs. 450

C)Rs. 900

D)Rs. 280

Explanation:-

DIRECTIONS for the question: Solve the following question and mark the best possible option.
Question No. : 37
Mr. Khan, a resident of city X, started by train for city Y, which is at a distance of 375 km from X, to meet his friend Thomas. As he
got into the train he looked at his watch and noted that it showed 4:00pm. When he reached station Y, Thomas met him and
looking at his watch and said As it is 8: 00 pm lets have our dinner. The next morning Khan started back for city X and Thomas
came along with him to send him off, When the train arrived , Thomas looked at his watch and said, It is 8: a.m. and the train has
arrived on time.When Khan reached X, his watch showed 2:00pm. The average speed of the train from Y to X is 100km/hr less
than the average speed of the train from X to Y. The watches of Mr. Khan and Thomas are both running at the constant speed i.e.
they neither lose time nor gain time.

What is the time shown by Khans watch, when the time shown by Thomas watch is 6:30 p.m.?
A)5: 30pm

B)5: 00pm

C)5: 40pm

D)Cannot be determined

Explanation:-
Let the speed of the train from X to Y be (s + 100) km/hr. Hence, the speed of the train from Y to X is s km/hr. All the answer
choices suggest that Khans watch trails Thomass say by x hour.

http://www.hitbullseye.com/frmTestView.aspx

19/59

7/7/2015

about:blank

4-x=2.5 x=1.5
When the time shown by Thomass watch is 6:30pm, the time shown by khans watch is 5:00 p.m.
DIRECTIONS for the question: Solve the following question and mark the best possible option.
Question No. : 38
Mr. Khan, a resident of city X, started by train for city Y, which is at a distance of 375 km from X, to meet his friend Thomas. As he
got into the train he looked at his watch and noted that it showed 4:00pm. When he reached station Y, Thomas met him and
looking at his watch and said As it is 8: 00 pm lets have our dinner. The next morning Khan started back for city X and Thomas
came along with him to send him off, When the train arrived , Thomas looked at his watch and said, It is 8: a.m. and the train has
arrived on time.When Khan reached X, his watch showed 2:00pm. The average speed of the train from Y to X is 100km/hr less
than the average speed of the train from X to Y. The watches of Mr. Khan and Thomas are both running at the constant speed i.e.
they neither lose time nor gain time.

What is the speed of the train from city X to city Y?


A)150 km/hr

B)140 km/hr

C)160 km/hr

D)Cannot be determined

Explanation:-
Let the speed of the train from X to Y be (s + 100) km/hr. Hence, the speed of the train from Y to X is s km/hr. All the answer
choices suggest that Khans watch trails Thomass say by x hour.

10s2 + 1000s = 750s + 37500


s2+25s-3750=0
(s 50) (s + 75)=0
s = 50 (Speed > 0)
The speed of the train from city X to city Y is s + 100 km/hr i.e. 150km/hr.

DIRECTIONS for the question: Solve the following question and mark the best possible option.
Question No. : 39
In the given figure, the circle is of unit radius and P is its centre. BGFE is a square. FB is extended to meet the circle at point D.

http://www.hitbullseye.com/frmTestView.aspx

20/59

7/7/2015

about:blank

What is the length of BF?


A)22

B)2

C)2

D)Cannot be determined

Explanation:-
By construction, we can observe the following:
Since, ABC = 90, hence, AD is a diameter of length 2 units.
Now, since, BGFE is a square, hence, GBO = 45
Hence, GBA = 45
Thus, GBA = ADB = 45 (Alternate Segment Theorem)

In s BDP and BGO,


PBD = OBE = GBO
BPD = BOG
Hence, they are similar.
PD : OB = BP : OG => OB: OG = PD: PB = 1 : 1
But, OB can be of any length. Thus, length of BF cant be determined.
DIRECTIONS for the question: Solve the following question and mark the best possible option.
Question No. : 40
In the given figure, the circle is of unit radius and P is its centre. BGFE is a square. FB is extended to meet the circle at point D.

http://www.hitbullseye.com/frmTestView.aspx

21/59

7/7/2015

about:blank

If ACB = 67.5 then what is the ratio of BC and CD?


A)2

B)1

C)1/2

D)2

Explanation:-
By construction, we can observe the following:
Since, ABC = 90, hence, AD is a diameter of length 2 units.
Now, since, BGFE is a square, hence, GBO = 45
Hence, GBA = 45
Thus, GBA = ADB = 45 (Alternate Segment Theorem)

We have
BP/BA = 1/2
Since BP = 1, hence BA=2 units
AD = 2
If ACB = 67.5 then BAC=22.5 and hence AC is angle bisector of BAD.
Hence, by angle bisector theorem, AB : AD = BC : CD = 1/2.
DIRECTIONS for the question: Solve the following question and mark the best possible option.
Question No. : 41
Rahim has Rs. 4,096 with him and he places bets 6 times. Each time he places a bet, he stakes one-fourth of the total money that
he has at the time of placing the bet. If he wins the bet, he gets back the sum of money he had staked and wins a sum equal to the
money he had staked and if he loses the bet, he loses the sum of money he had bet. If the probability of winning a betis 0.5 and it
is known that Rahim wins 3 bets and loses 3 of them, find the money left with Rahim at the end of all the bets.
A)Rs. 3,375

B)Rs. 721

C)Rs. 3072

http://www.hitbullseye.com/frmTestView.aspx

D)Cannot be determined
22/59

7/7/2015

about:blank

Explanation:-
Suppose Rahim has Rs. x initially and he loses a bet, the amount left with him will be Rs. 0.75x.Similarly, if Rahim has Rs. x initially
and he wins a bet, the amount now with him will be Rs. 1.25x. Thus, winning or losing only results in multiplying the amount by
1.25 or 0.75. Since he wins thrice, his original amount will be multiplied by 1.25 thrice and since he loses thrice, his original
amount will be multiplied by 0.75 thrice. Since multiplication is commutative, the order of winning and losing is not important.
Thus, inany case, finalamount left = 4096 0.753 1.253 = Rs. 3,375.

DIRECTIONS for the question: Solve the following question and mark the best possible option.
Question No. : 42
The XY-plane is marked on the ground in preparation for a sack race. The participants start from point P(0, 2) and finish the race at
point Q(10, 3). In hopping from P to Q, the participants must touch the X-axis exactly once. What point on the X-axis should a
participant touch so that the total distance travelled is the least?
A)(4, 0)

B)(6, 0)

C)(3, 0)

D)(5, 0)

Explanation:-

Take the reflection of point Q in the X-axis and call it Q (10, 3). Let the point at which the student touches x-axis be R. Then, by
symmetry, RQ = RQ and hence minimizing (PR + RQ) is same as minimizing (PR + RQ). It is evident that for (PR + RQ) to be
minimum, P, R and Q should be collinear. So (x, 0) lies on the line joining (0, 2) and (10, 3).

If the line touches x-axis then y becomes 0. Thus, x = 4 i.e. line touches x-axis at (4, 0)

DIRECTIONS for the question: Solve the following question and mark the best possible option.
Question No. : 43
In how many ways can 68068 be written as the difference of two squares?
A)8

B)10

C)12

D)16

Explanation:-

68068 = 22 7 11 13 17. Let 68068 = A2 B2 = (A + B)( A B), where A and B are whole numbers. Since the product is
even, at least one of (A + B) or (A B) must be even. To ensure this, both A and B must either be even or odd. Consequently, (A + B)
and (A B) are both even (A + B) = 2x and (A B) = 2y. So, x and y are any number of factors from among 7, 11, 13 and 17.
These can be chosen in 24 = 16 ways. If we select factors for x, the remaining factors will be y. A + B will always be greater than A
B. Half of the 16 possibilities will result in A + B < A B. So, the actual number of factors will be 16/2 = 8.Alternately, no of ways
will be total number of factors of (7 11 13 17) divided by 2. So answer is 8 ways.
DIRECTIONS for the question: Solve the following question and mark the best possible option.
Question No. : 44

A)4

B)-4

C)6

D)5

Explanation:-
http://www.hitbullseye.com/frmTestView.aspx

23/59

7/7/2015

about:blank

So the maximum value of the expression is 4.

DIRECTIONS for the question: Analyse the graph/s given below and answer the question that follows.
Question No. : 45

What was Company Ps expenditure on production in the year 2006?


A)Rs. 93,333

B)Rs. 284,000

C)Rs. 336,000

D)Rs. 38,888

Explanation:-
Company Ps total expenditure in 2006 = 2.8 360/100 lakh. Expenditure on production = 120/360 2.8360/100 lakh = 3.36
lakh.

http://www.hitbullseye.com/frmTestView.aspx

24/59

7/7/2015

about:blank

DIRECTIONS for the question: Analyse the graph/s given below and answer the question that follows.
Question No. : 46

In 2003, the total revenue earned by Company P was Rs. 8.4 lakhs and it had to pay a tax of 10.5% on revenue. If profit is
calculated as Revenue less expenses and tax, what was company Ps profit in 2003?
A)Rs. 590,700

B)Rs. 171,840

C)Rs. 571,800

D)Rs. 103,800

Explanation:-

http://www.hitbullseye.com/frmTestView.aspx

25/59

7/7/2015

about:blank

DIRECTIONS for the question: Analyse the graph/s given below and answer the question that follows.
Question No. : 47

What has been the increase in company Ps expenditure over the given period?
A)Rs. 220,000

B)Rs. 792,000

C)Rs. 824,000

D)Rs. 1,008,000

Explanation:-
Increase in advertising expenditure = Rs. (2.8 0.6)= Rs. 2.2 lakh. Increase in total expenditure = 2.2 360/100 = Rs. 7.92 lakhs.
DIRECTIONS for the question: Study the table/s given below and answer the question that follows.
Question No. : 48
Where India stands in Information Technology (1995-96)
Country

No. of
computers
per 1000
persons

No. of telephone lines


per 1000 persons

Software
Production
($mn)

IT
Spending
as % of
GDP

Aruba

300

1000

50,000

Austria

100

700

51,000

Bhutan

200

600

800

1.5

India

12

500

0.75

Chile

10

40

2700

0.4

China

12

2100

0.3

http://www.hitbullseye.com/frmTestView.aspx

26/59

7/7/2015

about:blank

If Chinas GDP is thrice as large as that of Indias GDP, then Chinas IT spending exceeds that of Indias by __
_____ percentage?

A)8 %

B)11 %

C)20 %

D)45 %

Explanation:-

DIRECTIONS for the question: Study the table/s given below and answer the question that follows.
Question No. : 49
Where India stands in Information Technology (1995-96)
Country

No. of
computers
per 1000
persons

No. of telephone lines


per 1000 persons

Software
Production
($mn)

IT
Spending
as % of
GDP

Aruba

300

1000

50,000

Austria

100

700

51,000

Bhutan

200

600

800

1.5

India

12

500

0.75

Chile

10

40

2700

0.4

China

12

2100

0.3

The ratio of telephone lines to computers is the smallest for which one of the following countries?
A)Aruba

B)Austria

C)Chile

D)Bhutan

Explanation:-
Aruba = 1000/300 = 3.33, Austria = 700/100 = 7, Chile = 40/10 = 4 and Bhutan = 600/200 = 3. Thus Bhutan has the lowest
ratio.
DIRECTIONS for the question: Solve the following question and mark the best possible option.
Question No. : 50
The letters M, N, O, P, Q, R and S, not necessarily in that order, stand for seven consecutive integers from 1 to 10. S is 4 less than
O, P is greater than M, Q is the middle term and M is as much less than Q as N is greater than S. If P has only one integer below it,
then the value of (R - S) will be:
A)5

B)6

C)4

D)2

Explanation:-
From the given condition: S = O 4, P > M and Q is the middle term. Q M = N S. We can make the diagram lowest to highest
(left to right)1 2 3 4 5 6 7 M P S Q R N O. So, R S = 2.

DIRECTIONS for the question: Each question has a set of 4 sequentially ordered statements which can be classified into

Facts: which deal with pieces of information that one has heard, seen, or read and which are open to discovery or verification (the
answer option indicates such a statement with an F).
Inferences: which are conclusions drawn about the unknown, on the basis of the known (the answer option indicates such a
statement with an I).
Judgements: which are opinions that imply approval or disapproval of persons, objects, situations and occurrences in the past, the
present or the future (the answer option indicates such a statement with a J).
Select the answer option that best describes the set of four statements.
http://www.hitbullseye.com/frmTestView.aspx

27/59

7/7/2015

about:blank

Question No. : 51

A. The Nanos body was completely redesigned twice and the engine three times.
B. Some concluded that the cost goals were too onerous and walked away.
C. In the longer term, the Nanos success will depend on how people react to the car and whether Mr Tata is right that there is a big gap
between what he calls the low end of the car market and the high end of the two-wheeler market
D. AT Kearney forecasts that sales of cars in India and the rest of Asia (excluding China) priced at $3,100-7,800 will reach 10.5 m by 2020.

A)FJJI

B)FIJI

C)IFJI

D)FJIF

Explanation:-
Option 4. Since statement A can be verified, it is a fact. In statement B, there is disapproval as the word onerous is used. In
statement C, an inference is made.
The author states that the success of Nano is dependant on the way people react to the car and whether Mr Tatas views are
correct.
Statement D is a fact because it can be verified. The forecast is made by AT Kearney but we can verify it by checking if they made
such a forecast. So, for us, it is a verifiable fact.
Hence option 4 is the best answer.

DIRECTIONS for the question: Each question has a set of 4 sequentially ordered statements which can be classified into

Facts: which deal with pieces of information that one has heard, seen, or read and which are open to discovery or verification (the
answer option indicates such a statement with an F).
Inferences: which are conclusions drawn about the unknown, on the basis of the known (the answer option indicates such a
statement with an I).
Judgements: which are opinions that imply approval or disapproval of persons, objects, situations and occurrences in the past, the
present or the future (the answer option indicates such a statement with a J).
Select the answer option that best describes the set of four statements.
Question No. : 52

A.Congestion, a mounting number of traffic accidents and worsening air pollution are the most obvious local problems
associated with rapidly increasing car ownership.
B.Attitudes to safety in most developing countries are more casual than in the rich West, but China has been rigorouslyenforcing
the wearing of seat belts, and consumers now expect at least a couple of airbags even in fairlybasic vehicles.
C. About 40% of Indias road traffic is carried on just 2% of its roads, most of which leave much to be desired.
D.What makes infrastructure investments in developing countries tricky is politics.
A)IFIF

B)FJJJ

C)JIJJ

D)FJFI

Explanation:-
Option 2. In statement 1, there is neither approval nor disapproval shown towards congestion. Also it can be verified. Hence it is a
fact.
In statement 2, there is a disapproval for the attitude to safety; hence it is a judgment.
In statement 3, disapproval is evident is the phrase leaves much to be desired.
In statement 4, disapproval is evident through the word tricky. Since approval or disapproval implies a judgement, the best
answer is option 2.

DIRECTIONS for the question: The question has five statements A, B, C, D and E, which when arranged logically form a coherent
sequence. From the options given choose the best sequence.
Question No. : 53

A. Without the distribution and manufacturing efficiencies of the modern age, without the toll-free numbers and express delivery
and bar codes and scanners and, above all, computer, the choices would not be multiplying like this.
B. Everywhere you turn, someone is offering advice on things like which of the thousands of mutual funds to buy.
C. Consumer psychologists say this sea of choices is driving us bonkers.
D. Or the right MBA program from among hundreds of business schools.
E. Superior performance in this competitive world is all about mastering business basics.
A)BDCAE

B)BEACD

C)ACEDB

http://www.hitbullseye.com/frmTestView.aspx

D)DBCAE

28/59

7/7/2015

about:blank

Explanation:-
Option 1.this sea of choices of C refers to the mutual funds (B) and MBA programs (D) giving us the sequence BDC. From the
choices would not multiply like this, we can infer that A will follow BDC. E provides a prescription to survive in this competitive
world.

DIRECTIONS for the question: Each question has five statements A, B, C, D and E, which when arranged logically form a coherent
sequence. From the options given choose the best sequence.
Question No. : 54

A. A large majority of these school dropouts has limited access to vocational training or any form of skill development program
as there are just about 2.5 to 3 million seats for such education.
B. For instance, hardly 5% of 20-24 year olds of our workforce has received any form of vocational training compared to 60-80%
in developed countries.
C. The level of educated among the existing workforce is low and on the other hand the educated without professional skills is
high.
D. Thus, in view of the working age population which is expected to grow by more than 47 million by 2020, equipping this evergrowing workforce with appropriate skills and knowledge and harnessing their potential as human capital is the need of the
hour.
E.Significantly, while 200 million students enrol for primary education, only 20 million are able to finish class 12.
A)DCBAE

B)EABDC

C)CBEAD

D)CEABD

Explanation:-
Option 3. D is evidently the conclusion of the passage. The large majority of school dropouts of A is derived from of the 200m
primary enrolments only 20m complete class 12 of E. Similarly, hardly 5% of 20-24 year olds of B is an explanation to
statement C. Of the pairs, CB and EA, CB logically precedes EA giving us CBEAD.

DIRECTIONS for the question: A part of the sentence in each sentence has been omitted and four alternate ways of phrasing the
blank text have been given in the options below. Choose the option that provides the most meaningful sentence.
Question No. : 55

Climate change has been ____________ in the developing countries.


A)identified as a new battleground, as an elaborate conspiracy by developed world to throttle growth
B)identified as the new battleground, as an elaborate conspiracy by the developed world for throttling the growth
C)identified as the new battleground, as the elaborate conspiracy by the developed world to throttle growth
D)identified as the new battleground, as an elaborate conspiracy by the developed world to throttle growth
Explanation:-
Option 4. We use the, the definite article, initially to stress Climate change and then an, the indefinite article, to continue.

DIRECTIONS for the question: A part of the sentence in each sentence has been omitted and four alternate ways of phrasing the
blank text have been given in the options below. Choose the option that provides the most meaningful sentence.
Question No. : 56

So strong is this pressure that ___________________.


A)Minister for the Environment has made commitment to Parliament that India would not accept any binding emission
reductions at the forthcoming Copenhagen climate change summit.
B)the Minister for Environment has committed in Parliament that Indians would not accept any binding emission reductions at
the forth coming Copenhagen climate change summit.
C)the Minister for Environment has committed to Parliament that India would not accept any binding emission reductions at
the forthcoming Copenhagen climate change summit.
D)Minister for the Environment has committed to Parliament that India would not accept any binding emission reductions at
the forthcoming Copenhagen climate change summit.
Explanation:-
Option 3. We need 'the' before 'minister'. The minister has committed to Parliament that India (not Indians as the word India
http://www.hitbullseye.com/frmTestView.aspx

29/59

7/7/2015

about:blank

represents the nation).

DIRECTIONS for the question: In each of the following paragraph, a part of the paragraph is left unfinished. Beneath the
paragraph, four different ways of completing the paragraph are indicated. Choose the best alternative amongst the four.
Question No. : 57

A larger brain are fine endowments, but they presented a major difficulty: that of being born. A child with a large-size brain could
not possibly be born through its mothers birth passage and pelvis. The answer, from the point of view of evolution, was a
compromise: the female pelvis broadened somewhat to accommodate the larger head of the foetus; the infants were born with
considerably less than their full brain size and development. In chimpanzees, to give a rough comparison, babies are born with 65
percent of their adult capacity; in the australopithecines the percentage was about 50 percent ; in modern humans the figure is
about 25 percent. That is, a modern baby has a brain which is about one-quarter the size of the brain that it will eventually
develop. A related fact of significance is that in contrast to the young of other animals, a human baby is immature,
underdeveloped, and in fact, as well all know, entirely hopeless. This evolutionary development has had two important
consequences. One is the changed shape and functioning of the female the broadened hips and rather rolling gait that is typical
of the human female are the direct results of the need to accommodate the enlarged brain. This change brought to females a
disadvantage in walking and running.
A)Thus the childs dependency on the mother was prolonged resulting in better bonding.
B)Thus the adult human looks more like a young female chimpanzee than the adult chimpanzee.
C)The young female apes are brighter brained and larger headed than adult humans.
D)Thus when hunting swift animals the men were the ones who did the chasing while women tended to other jobs
Explanation:-
Option 4. We are looking for a continuation of the passage. Option 4 correctly takes the idea forward by stating that the reason
why men hunted swift animals was because of the disadvantage that women had in the walking and running.
In option 1, we cannot say that the disadvantage in walking and running caused the childs dependency on the mother. The brain
and head size is being compared and not the looks of the chimpanzee vis--vis humans.
The passage states that chimpanzees are born with 65% of their full capacity but it does not mention whether that makes them
better brained

DIRECTIONS for the question: In each of the following paragraph, a part of the paragraph is left unfinished. Beneath the
paragraph, four different ways of completing the paragraph are indicated. Choose the best alternative amongst the four.
Question No. : 58
It didnt matter that a company was profitable; if investors could get higher profits and higher stock prices elsewhere, the company
was still under the gun. Nor did it matter that a product was relatively cheap and of fair quality, if consumers could get better deals
elsewhere. Workers who inhabited the local service economy faced a different challenge from their counterparts in big industry.
Their jobs werent in danger of disappearing. They couldnt be outsourced abroad and most wouldnt be automated. In fact, the
number of localservice jobs kept growing.
A)Significantly, most of these service jobs were not unionized
B)The real problem was that these jobs tended to pay very low wages, rarely included any benefits, and provided little chance

for advancement.
C)Their unions were interested in preserving good wages and benefits of its current members
D)The service unions saw their mission less as preserving good jobs in danger of disappearing and more as boosting the
prospects of people trapped in poor ones.
Explanation:-
Option 2. The paragraph states that workers who inhabited the local service economy faced a different challenge from their
counterparts in the big industry and further states the things which do not pose a problem. Then the next sentence would be the
thing that poses the problem which is of course the low wages, no benefits and little change of advancement. Hence option 2.

DIRECTIONS for the question: Each sentence contains pairs of words highlighted and labelled A and B. Select the option which gives the
best word choice for each pair.

Question No. : 59

Rural electrification and publicly funded (A)/sponsored (B) optical fibre networks in all parts of the country are complementary
(A)/complimentary (B) investments that the government must make, to remove the present stylist (A)/elitist (B) bias in computer access.
As online testing becomes gratuitous (A)/ubiquitous (B), felicity (A)/facility (B) with the hardware should not be a differentiating factor for
http://www.hitbullseye.com/frmTestView.aspx

30/59

7/7/2015

about:blank

those being tested.

A)ABBAB

B)ABBBB

C)BABBA

D)AABBA

Explanation:-
Option 4. We would use funded and not sponsored after publicly.
Complementary forming a complement which means to provide something felt to be lacking or needed. Here this word is needed
because in order to remove the bias we need complement and not compliment.
Gratuitous means given, done, bestowed, or obtained without charge or payment; free; voluntary. Here the writer wants to say that the
electrification and optical fibre networks will be needed when the testing becomes online everywhere, hence ubiquitous. Felicity: An

instance of appropriate and pleasing manner or style.

DIRECTIONS for the question: Each sentence contains pairs of words highlighted and labelled A and B. Select the option which gives the
best word choice for each pair.

Question No. : 60

Personality is a term with many varying (A)/differing (B) connotations (A)/nuances (B), depending on the gist (A)/context (B) of usage.
In medical and psychological argot (A)/parlance (B), personality is used to denote those characteristics of a person that account for
consistent patterns of feeling, thinking and behaving; unique and enduring (A)/endearing (B) patterns of behaviour and emotional
response which make us distinct individuals.

A)BAABB

B)BAABA

C)AABBA

D)ABAAB

Explanation:-
Option 3. Personality means different things when used in different contexts. Hence we need varying as it shows diversity and connotations
because we are talking about differences in meanings. Parlance means a way or manner of speaking; vernacular; idiom: legal parlance.
Enduring means long lasting or permanent. And Endearing means tending to make dear or beloved.

Question No. : 61
DIRECTIONS for the question: The passages given below are followed by a set of question. Choose the most appropriate answer
to each question.
PASSAGE I
I do not pretend that the development of trust in leadership is a science or something that may be perfected far from it. And I am
not suggesting that the development of genuine humility, and finally trust, in leadership is by any means easy. It is the hardest
thing the human creature called man can do. Anyone suggesting that he is, in fact, a person or leader of humility, moves farther
from it.
Warren Bennis argues that leaders rarely fail because of technical incompetence. Instead, where leaders predominantly fail is
weakness on the softer issues such as people skills, taste, judgment, and above all, character.
The most compelling leaders lead and keep their trust when they start with a proper view of themselves. By embracing this
essential humility, leaders will not only influence and lead, but will transform the lives of those around them, reproducing
leadership in others. This essence is what Professor Lewis would have referred to as mereness.
Applied to leadership, this mereness occurs, first, when leaders develop a core understanding of their humanity; second, when
they understand their depraved nature; and third, when leaders finally grasp that the purpose of leadership is not leadership itself.
When this mereness is revealed in leaders, they build trust. This, is turn, properly allows them to serve others.
Whether you hold a materialistic view of the universe (that matter and space have always existed and nobody know why) or the
theistic view (that there is something behind the universe that has a mind and a conscious purpose) we are in fact alike. Nothing
like stating the obvious, but it must be stated in leadership. It is the foundation.
Even Sigmund Freud, who rejected a theistic view of the universe in favour of a materialistic or scientific one, still seemed to
acknowledge some kind of unexplainable force in the universe. Freud experienced strange, secret longings that he described as
sechsucht. C.S. Lewis characterized his sechsucht as an unsatisfied desire which is itself more desirable than any other
satisfaction.
Whether we are born in poverty or into wealth; whether we are born in Beverly Hills or in Calcutta; whether we are born with
disabilities or not; whether we are born white, yellow, brown, or black; we are, in terms of these longings, and our human nature,
intrinsically alike.
In terms of pain regardless of our backgrounds, lifestyles, and worldviews we all have, like the apostle Paul, a thorn
somewhere in our flesh. While some acknowledge those thorns, others bury them deep within their souls not only to conceal
them from others, but also to pretend that they do not exist. Do not deny for a minute that they are not real. We are the creatures
called man.
Moreover, there are certain decent moral behaviours to which we all adhere. There are, in fact, laws of decent behaviour that
without formal moral or religious instruction ought to naturally govern our behaviour.
http://www.hitbullseye.com/frmTestView.aspx

31/59

7/7/2015

about:blank

Men have differed as regards what people who ought to be unselfish to whether it was your family, or your fellow countrymen,
or everyone, wrote Lewis. But they have always agreed that you ought not to put yourself first. Selfishness has never been
admired.
Look at the corporate life: one of the common business practices over the last decade has been to manipulate accounting rules in
order to maximize the earnings of public companies. Enrons former treasurer Jeffrey McMahon declared that Enron decided to
obey only the accounting rules that got them the results they wanted. Inherent in his argument is the insinuation that rules may
have been broken, but until he is caught or told otherwise, he will continue to practice.
While other energy companies also practised such accounting, it didnt make Enrons use of mark to market, and other creative
accounting gimmicks (such as hiding debt in special-purpose entities), any more correct. Some of the blame for the corporate
fraud of the 90s must be placed at the feet of regulators who made changes in the method of accounting standards, the culture of
Wall Street that demanded aggressive earnings growth, and executives whose compensation targets were tied to the price of their
own personal options.
Blame could be spread far and wide, but the fact remains that at some point some leader (not accounting rule) had to make a
conscious decision to inflate earnings. Whether other competitors were doing it or not or whether the accounting standards were
loose enough to enable them, most leaders knew such actions were questionable, if not outright wrong.
Excerpted from: Trust by Les T. Csorba.
The primary focus of the passage is on
A)Humility, morality and integrity B)Leadership, humility and ratiocination
D)Humility, leadership and moral behaviour

C)Humanity, materialism and rationalization

Explanation:-
Option 4. The author initially says that humility is one of the important factors on which successful leadership depends as leaders
are able to serve others properly. He then goes on to talk about decent moral behaviours and by giving the example of Enron says
that systems and rules cannot be apportioned blame as ultimately it is a person taking them.

Question No. : 62
DIRECTIONS for the question: The passages given below are followed by a set of question. Choose the most appropriate answer to
each question.
PASSAGE I
I do not pretend that the development of trust in leadership is a science or something that may be perfected far from it. And I am not
suggesting that the development of genuine humility, and finally trust, in leadership is by any means easy. It is the hardest thing the human
creature called man can do. Anyone suggesting that he is, in fact, a person or leader of humility, moves farther from it.
Warren Bennis argues that leaders rarely fail because of technical incompetence. Instead, where leaders predominantly fail is weakness on
the softer issues such as people skills, taste, judgment, and above all, character.
The most compelling leaders lead and keep their trust when they start with a proper view of themselves. By embracing this essential
humility, leaders will not only influence and lead, but will transform the lives of those around them, reproducing leadership in others. This
essence is what Professor Lewis would have referred to as mereness.
Applied to leadership, this mereness occurs, first, when leaders develop a core understanding of their humanity; second, when they
understand their depraved nature; and third, when leaders finally grasp that the purpose of leadership is not leadership itself. When this
mereness is revealed in leaders, they build trust. This, is turn, properly allows them to serve others.
Whether you hold a materialistic view of the universe (that matter and space have always existed and nobody know why) or the theistic
view (that there is something behind the universe that has a mind and a conscious purpose) we are in fact alike. Nothing like stating the
obvious, but it must be stated in leadership. It is the foundation.
Even Sigmund Freud, who rejected a theistic view of the universe in favour of a materialistic or scientific one, still seemed to
acknowledge some kind of unexplainable force in the universe. Freud experienced strange, secret longings that he described as
sechsucht. C.S. Lewis characterized his sechsucht as an unsatisfied desire which is itself more desirable than any other satisfaction.
Whether we are born in poverty or into wealth; whether we are born in Beverly Hills or in Calcutta; whether we are born with disabilities or
not; whether we are born white, yellow, brown, or black; we are, in terms of these longings, and our human nature, intrinsically alike.
In terms of pain regardless of our backgrounds, lifestyles, and worldviews we all have, like the apostle Paul, a thorn somewhere in our
flesh. While some acknowledge those thorns, others bury them deep within their souls not only to conceal them from others, but also to
pretend that they do not exist. Do not deny for a minute that they are not real. We are the creatures called man.
Moreover, there are certain decent moral behaviours to which we all adhere. There are, in fact, laws of decent behaviour that without formal
moral or religious instruction ought to naturally govern our behaviour.
Men have differed as regards what people who ought to be unselfish to whether it was your family, or your fellow countrymen, or
everyone, wrote Lewis. But they have always agreed that you ought not to put yourself first. Selfishness has never been admired.
Look at the corporate life: one of the common business practices over the last decade has been to manipulate accounting rules in order to
maximize the earnings of public companies. Enrons former treasurer Jeffrey McMahon declared that Enron decided to obey only the
accounting rules that got them the results they wanted. Inherent in his argument is the insinuation that rules may have been broken, but until
he is caught or told otherwise, he will continue to practice.
While other energy companies also practised such accounting, it didnt make Enrons use of mark to market, and other creative
accounting gimmicks (such as hiding debt in special-purpose entities), any more correct. Some of the blame for the corporate fraud of the
90s must be placed at the feet of regulators who made changes in the method of accounting standards, the culture of Wall Street that
http://www.hitbullseye.com/frmTestView.aspx

32/59

7/7/2015

about:blank

demanded aggressive earnings growth, and executives whose compensation targets were tied to the price of their own personal options.
Blame could be spread far and wide, but the fact remains that at some point some leader (not accounting rule) had to make a conscious
decision to inflate earnings. Whether other competitors were doing it or not or whether the accounting standards were loose enough to
enable them, most leaders knew such actions were questionable, if not outright wrong.
Excerpted from: Trust by Les T. Csorba.
By giving example of Enrons Jeffrey McMahon, the author wants to highlight the fact that

A)In the 1990s most large corporations followed disputable accountancy practices and hence underscoring only Enron is incorrect.
B)Questionable actions by the leader cannot be justified even in adverse circumstances or conditions
C)At that time, the culture of Wall Street made it mandatory for leaders to follow aggressive tactics
D)If not caught, a company should follow improper practices for financial gains
Explanation:-
Option 2. The author states that Jeffery insinuated that if not caught malpractices were okay. The author does not agree to this and he says
that it is leaders who made decisions to inflate earnings and hence the system cannot be held at ransom. The author does not state the
example of Jeffrey to state that the Enron corporation was being made a scapegoat when all others we doing the same. Wall Street culture
demanded aggressive earnings growth but that didnt mean it was mandatory i.e. absolutely necessary. The author does not agree with
option 4.

Question No. : 63
DIRECTIONS for the question: The passages given below are followed by a set of question. Choose the most appropriate answer
to each question.
PASSAGE I
I do not pretend that the development of trust in leadership is a science or something that may be perfected far from it. And I am
not suggesting that the development of genuine humility, and finally trust, in leadership is by any means easy. It is the hardest
thing the human creature called man can do. Anyone suggesting that he is, in fact, a person or leader of humility, moves farther
from it.
Warren Bennis argues that leaders rarely fail because of technical incompetence. Instead, where leaders predominantly fail is
weakness on the softer issues such as people skills, taste, judgment, and above all, character.
The most compelling leaders lead and keep their trust when they start with a proper view of themselves. By embracing this
essential humility, leaders will not only influence and lead, but will transform the lives of those around them, reproducing
leadership in others. This essence is what Professor Lewis would have referred to as mereness.
Applied to leadership, this mereness occurs, first, when leaders develop a core understanding of their humanity; second, when
they understand their depraved nature; and third, when leaders finally grasp that the purpose of leadership is not leadership itself.
When this mereness is revealed in leaders, they build trust. This, is turn, properly allows them to serve others.
Whether you hold a materialistic view of the universe (that matter and space have always existed and nobody know why) or the
theistic view (that there is something behind the universe that has a mind and a conscious purpose) we are in fact alike. Nothing
like stating the obvious, but it must be stated in leadership. It is the foundation.
Even Sigmund Freud, who rejected a theistic view of the universe in favour of a materialistic or scientific one, still seemed to
acknowledge some kind of unexplainable force in the universe. Freud experienced strange, secret longings that he described as
sechsucht. C.S. Lewis characterized his sechsucht as an unsatisfied desire which is itself more desirable than any other
satisfaction.
Whether we are born in poverty or into wealth; whether we are born in Beverly Hills or in Calcutta; whether we are born with
disabilities or not; whether we are born white, yellow, brown, or black; we are, in terms of these longings, and our human nature,
intrinsically alike.
In terms of pain regardless of our backgrounds, lifestyles, and worldviews we all have, like the apostle Paul, a thorn
somewhere in our flesh. While some acknowledge those thorns, others bury them deep within their souls not only to conceal
them from others, but also to pretend that they do not exist. Do not deny for a minute that they are not real. We are the creatures
called man.
Moreover, there are certain decent moral behaviours to which we all adhere. There are, in fact, laws of decent behaviour that
without formal moral or religious instruction ought to naturally govern our behaviour.
Men have differed as regards what people who ought to be unselfish to whether it was your family, or your fellow countrymen,
or everyone, wrote Lewis. But they have always agreed that you ought not to put yourself first. Selfishness has never been
admired.
Look at the corporate life: one of the common business practices over the last decade has been to manipulate accounting rules in
order to maximize the earnings of public companies. Enrons former treasurer Jeffrey McMahon declared that Enron decided to
obey only the accounting rules that got them the results they wanted. Inherent in his argument is the insinuation that rules may
have been broken, but until he is caught or told otherwise, he will continue to practice.
While other energy companies also practised such accounting, it didnt make Enrons use of mark to market, and other creative
http://www.hitbullseye.com/frmTestView.aspx

33/59

7/7/2015

about:blank

accounting gimmicks (such as hiding debt in special-purpose entities), any more correct. Some of the blame for the corporate
fraud of the 90s must be placed at the feet of regulators who made changes in the method of accounting standards, the culture of
Wall Street that demanded aggressive earnings growth, and executives whose compensation targets were tied to the price of their
own personal options.
Blame could be spread far and wide, but the fact remains that at some point some leader (not accounting rule) had to make a
conscious decision to inflate earnings. Whether other competitors were doing it or not or whether the accounting standards were
loose enough to enable them, most leaders knew such actions were questionable, if not outright wrong.
Excerpted from: Trust by Les T. Csorba.
According the passage we can infer all of the following, except
A)Good leaders are interested in their followers and the organisation.
B)Leaders performance depends upon personal character C)Humility is rarely present in a person who says he possesses it.
D)Mereness in a leader signifies them being selfless or servile in their disposition.
Explanation:-
Option 4. The author terms mereness as a quality which leaders need to transform the lives of those around them.
Hence a leader who is servile i.e. having a slave like or submissive disposition is hardly going to be able to lead.

Question No. : 64
DIRECTIONS for the question: The passage given below are followed by a set of question. Choose the most appropriate answer to each
question.
PASSAGE II
The drive to advance I think you have to ask exactly what that means. If you mean a drive to produce more, well, who wants it? Is that
necessarily the right thing to do? Its obvious. In fact, in many areas its probably the wrong thing to do maybe its a good thing that there
wouldnt be the same drive to produce. People have to be driven to have certain wants in our system why? Why not leave them alone so
they can just be happy, do their things?
Whatever drive there is ought to be internal. So take a look at kids: theyre creative, they explore, they want to try new things. You take a
one-year old kid, hes crawling fine, he can get anywhere across the room he likes really fast, so fast his parents have to run to keep him
from knocking everything down all of a sudden he gets up and starts walking. Hes terrible at walking: he walks one step and he falls on
his face, and if he wants to really get somewhere hes going to crawl. So why do kids start walking? Well, they just want to do a new thing,
thats the way people are built. Were built to want to do new things, even if theyre not efficient, even if theyre harmful, even if you get hurt
and I dont think that ever stops.
People want to explore, we want to press our capacities to their limits, we want to appreciate what we can. But the joy of creation is
something very few people get the opportunity to have in our society: artists get to have it, craftspeople have it, scientists. And if youve been
lucky enough to have had that opportunity, you know its quite an experience and it doesnt have to be discovering Einsteins theory of
relativity: anybody can have the pleasure, even by seeing what other people have done. For instance, if you read even a simple
mathematical proof like the Pythagorean Theorem, what you study in tenth grade, and finally figure out what its all about, thats exciting
My God, I never understood that before. Okay, thats creativity, even though somebody else proved it thousand years ago.
You just keep being struck by the marvels of what youre discovering, and youre discovering it, even though somebody else did it already.
Then if you can ever add a little bit to whats already known alright, thats very exciting. And I think the same thing is true of a person who
builds a boat: I dont see why its fundamentally any different I mean, I wish I could do that; I cant, I cant imagine doing it.
Well, I think the people should be able to live in a society where they can exercise these kinds of internal drives and develop their capacities
freely instead of being forced into the narrow range of options that are available to most people in the world now. And by that, I mean not
only options that are objectively available, but also options that are subjectively available like, how are people allowed to think, how are
they able to think? Remember, there are all kinds of ways of thinking that are cut off from us in our society not because were incapable of
them, but because various blockages have been developed and imposed to prevent people from thinking in those ways.
So, I think what has to happen is, other options have to be opened up to people both subjectively, and in fact concretely: meaning you can
do something about them without great suffering. And thats one of the main purposes of socialism, I think: to reach a point where people
have the opportunity to decide freely for themselves what their needs are, and not just have the choices forced on them by some arbitrary
system of power.
Excerpted from: Understanding Power by Noam Chomsky.
Through the example of a kid starting to walk, the author wants to point out that

A)External forces play a major role in determining what a person should or would do
B)Activities need not always be done with the objective of learning something
C)Each of us has an inherent urge to explore and use our capacities to the fullest
D)Unless you fail in your initial endeavours you cannot progress
Explanation:-
Option 3. Each one of us has an inherent drive in us and we want to use our capacities to fulfil that drive. The author further states that
people dont need to be driven and can be self motivated. The desire to explore i.e. venture into something new is immanent in us.

Question No. : 65
http://www.hitbullseye.com/frmTestView.aspx

34/59

7/7/2015

about:blank

DIRECTIONS for the question: The passage given below are followed by a set of question. Choose the most appropriate answer to each
question.
PASSAGE II
The drive to advance I think you have to ask exactly what that means. If you mean a drive to produce more, well, who wants it? Is that
necessarily the right thing to do? Its obvious. In fact, in many areas its probably the wrong thing to do maybe its a good thing that there
wouldnt be the same drive to produce. People have to be driven to have certain wants in our system why? Why not leave them alone so
they can just be happy, do their things?
Whatever drive there is ought to be internal. So take a look at kids: theyre creative, they explore, they want to try new things. You take a
one-year old kid, hes crawling fine, he can get anywhere across the room he likes really fast, so fast his parents have to run to keep him
from knocking everything down all of a sudden he gets up and starts walking. Hes terrible at walking: he walks one step and he falls on
his face, and if he wants to really get somewhere hes going to crawl. So why do kids start walking? Well, they just want to do a new thing,
thats the way people are built. Were built to want to do new things, even if theyre not efficient, even if theyre harmful, even if you get hurt
and I dont think that ever stops.
People want to explore, we want to press our capacities to their limits, we want to appreciate what we can. But the joy of creation is
something very few people get the opportunity to have in our society: artists get to have it, craftspeople have it, scientists. And if youve been
lucky enough to have had that opportunity, you know its quite an experience and it doesnt have to be discovering Einsteins theory of
relativity: anybody can have the pleasure, even by seeing what other people have done. For instance, if you read even a simple
mathematical proof like the Pythagorean Theorem, what you study in tenth grade, and finally figure out what its all about, thats exciting
My God, I never understood that before. Okay, thats creativity, even though somebody else proved it thousand years ago.
You just keep being struck by the marvels of what youre discovering, and youre discovering it, even though somebody else did it already.
Then if you can ever add a little bit to whats already known alright, thats very exciting. And I think the same thing is true of a person who
builds a boat: I dont see why its fundamentally any different I mean, I wish I could do that; I cant, I cant imagine doing it.
Well, I think the people should be able to live in a society where they can exercise these kinds of internal drives and develop their capacities
freely instead of being forced into the narrow range of options that are available to most people in the world now. And by that, I mean not
only options that are objectively available, but also options that are subjectively available like, how are people allowed to think, how are
they able to think? Remember, there are all kinds of ways of thinking that are cut off from us in our society not because were incapable of
them, but because various blockages have been developed and imposed to prevent people from thinking in those ways.
So, I think what has to happen is, other options have to be opened up to people both subjectively, and in fact concretely: meaning you can
do something about them without great suffering. And thats one of the main purposes of socialism, I think: to reach a point where people
have the opportunity to decide freely for themselves what their needs are, and not just have the choices forced on them by some arbitrary
system of power.
Excerpted from: Understanding Power by Noam Chomsky.
The joy of creation can be realized by

A)Experiencing vicariously through the discoveries of others B)Only an elite group of artists, craftsmen and probably scientists
C)Only through first hand experience of creating something original D)Some people who can utilise their capacities to the fullest
Explanation:-
Option 1. The author states that one can experience the joy of creation even through an indirect way (example of Pythagorean Theorem).
Option 3 is incorrect as the joy of creation does not have to be through the first hand experience of creating something new or original. The
word ONLY makes option 2 incorrect. The appreciation is of creation and not of the capacities. So option 4 is incorrect.

Question No. : 66
DIRECTIONS for the question: The passage given below are followed by a set of question. Choose the most appropriate answer to each
question.
PASSAGE II
The drive to advance I think you have to ask exactly what that means. If you mean a drive to produce more, well, who wants it? Is that
necessarily the right thing to do? Its obvious. In fact, in many areas its probably the wrong thing to do maybe its a good thing that there
wouldnt be the same drive to produce. People have to be driven to have certain wants in our system why? Why not leave them alone so
they can just be happy, do their things?
Whatever drive there is ought to be internal. So take a look at kids: theyre creative, they explore, they want to try new things. You take a
one-year old kid, hes crawling fine, he can get anywhere across the room he likes really fast, so fast his parents have to run to keep him
from knocking everything down all of a sudden he gets up and starts walking. Hes terrible at walking: he walks one step and he falls on
his face, and if he wants to really get somewhere hes going to crawl. So why do kids start walking? Well, they just want to do a new thing,
thats the way people are built. Were built to want to do new things, even if theyre not efficient, even if theyre harmful, even if you get hurt
and I dont think that ever stops.
People want to explore, we want to press our capacities to their limits, we want to appreciate what we can. But the joy of creation is
something very few people get the opportunity to have in our society: artists get to have it, craftspeople have it, scientists. And if youve been
lucky enough to have had that opportunity, you know its quite an experience and it doesnt have to be discovering Einsteins theory of
relativity: anybody can have the pleasure, even by seeing what other people have done. For instance, if you read even a simple
mathematical proof like the Pythagorean Theorem, what you study in tenth grade, and finally figure out what its all about, thats exciting
My God, I never understood that before. Okay, thats creativity, even though somebody else proved it thousand years ago.
You just keep being struck by the marvels of what youre discovering, and youre discovering it, even though somebody else did it already.
Then if you can ever add a little bit to whats already known alright, thats very exciting. And I think the same thing is true of a person who
builds a boat: I dont see why its fundamentally any different I mean, I wish I could do that; I cant, I cant imagine doing it.
Well, I think the people should be able to live in a society where they can exercise these kinds of internal drives and develop their capacities
freely instead of being forced into the narrow range of options that are available to most people in the world now. And by that, I mean not
only options that are objectively available, but also options that are subjectively available like, how are people allowed to think, how are
http://www.hitbullseye.com/frmTestView.aspx

35/59

7/7/2015

about:blank

they able to think? Remember, there are all kinds of ways of thinking that are cut off from us in our society not because were incapable of
them, but because various blockages have been developed and imposed to prevent people from thinking in those ways.
So, I think what has to happen is, other options have to be opened up to people both subjectively, and in fact concretely: meaning you can
do something about them without great suffering. And thats one of the main purposes of socialism, I think: to reach a point where people
have the opportunity to decide freely for themselves what their needs are, and not just have the choices forced on them by some arbitrary
system of power.
Excerpted from: Understanding Power by Noam Chomsky.
According to the passage all of the following are untrue, EXCEPT

A)The author states that freedom to choose things hampers progress


B)Indoctrination is like being shackled, which prevents people from thinking on their own.
C)People experience joy only when they are able to discover or invent something new.
D)In order to think subjectively, options need to be opened up to people
Explanation:-
Option 2. The author states that there are all kinds of ways of thinking that are cut off from us in our society. Instead of being forced to
choose from the choices given each one should be given freedom to make the choices in the first place. Option 3 is incorrect because joy
can be felt through the discoveries of others too.

Question No. : 67
DIRECTIONS for the question: The passage given below are followed by a set of question. Choose the most appropriate answer to each
question.
PASSAGE II
The drive to advance I think you have to ask exactly what that means. If you mean a drive to produce more, well, who wants it? Is that
necessarily the right thing to do? Its obvious. In fact, in many areas its probably the wrong thing to do maybe its a good thing that there
wouldnt be the same drive to produce. People have to be driven to have certain wants in our system why? Why not leave them alone so
they can just be happy, do their things?
Whatever drive there is ought to be internal. So take a look at kids: theyre creative, they explore, they want to try new things. You take a
one-year old kid, hes crawling fine, he can get anywhere across the room he likes really fast, so fast his parents have to run to keep him
from knocking everything down all of a sudden he gets up and starts walking. Hes terrible at walking: he walks one step and he falls on
his face, and if he wants to really get somewhere hes going to crawl. So why do kids start walking? Well, they just want to do a new thing,
thats the way people are built. Were built to want to do new things, even if theyre not efficient, even if theyre harmful, even if you get hurt
and I dont think that ever stops.
People want to explore, we want to press our capacities to their limits, we want to appreciate what we can. But the joy of creation is
something very few people get the opportunity to have in our society: artists get to have it, craftspeople have it, scientists. And if youve been
lucky enough to have had that opportunity, you know its quite an experience and it doesnt have to be discovering Einsteins theory of
relativity: anybody can have the pleasure, even by seeing what other people have done. For instance, if you read even a simple
mathematical proof like the Pythagorean Theorem, what you study in tenth grade, and finally figure out what its all about, thats exciting
My God, I never understood that before. Okay, thats creativity, even though somebody else proved it thousand years ago.
You just keep being struck by the marvels of what youre discovering, and youre discovering it, even though somebody else did it already.
Then if you can ever add a little bit to whats already known alright, thats very exciting. And I think the same thing is true of a person who
builds a boat: I dont see why its fundamentally any different I mean, I wish I could do that; I cant, I cant imagine doing it.
Well, I think the people should be able to live in a society where they can exercise these kinds of internal drives and develop their capacities
freely instead of being forced into the narrow range of options that are available to most people in the world now. And by that, I mean not
only options that are objectively available, but also options that are subjectively available like, how are people allowed to think, how are
they able to think? Remember, there are all kinds of ways of thinking that are cut off from us in our society not because were incapable of
them, but because various blockages have been developed and imposed to prevent people from thinking in those ways.
So, I think what has to happen is, other options have to be opened up to people both subjectively, and in fact concretely: meaning you can
do something about them without great suffering. And thats one of the main purposes of socialism, I think: to reach a point where people
have the opportunity to decide freely for themselves what their needs are, and not just have the choices forced on them by some arbitrary
system of power.
Excerpted from: Understanding Power by Noam Chomsky.
The tone of the passage is

A)Sarcastic and derogatory

B)Disapproving yet insightful

C)Pedantic yet approving

D)Authoritative and placating

Explanation:-
Option 2. The author is disapproving of people being driven and wants people to make their own choices. He also thinks that creativity can
be experienced by everyone directly or vicariously. The tone is quite insightful. He is not being sarcastic or belittling anyone.
The issue he deals with is one of great importance as he is talking about drive and the choices available which are important to man. He is
not calming or appeasing anyone. He wants the people to be given freedom to make their own choices and choose from them.

Question No. : 68
DIRECTIONS for the question: The passage given below are followed by a set of question. Choose the most appropriate answer to each
question.
http://www.hitbullseye.com/frmTestView.aspx

36/59

7/7/2015

about:blank

PASSAGE III
Financial speculators dont want growth: what they want is stable currencies, meaning no growth. In fact, the business press talks openly
now about the threat of too much growth, the threat of too much employment: theyre perfectly open about all of this, to one another. And
the reason for it is, people who speculate against currencies are afraid of inflation because it decreases the value of their money, so
therefore its a big threat to them. Currency speculators dont like that, so if they see signs of simulative economic policies or anything that
may bring economic growth, theyll just take their capital out of that companies economy and even a slight withdrawal of that sort can
easily trigger a recession in those countries.
So whats happened as a result of all this is a big drift internationally towards low-growth, low-wage, high-profit economies because
national governments trying to make economic and social policy decisions just have very little leeway to do so by now, or else their
economies will be wrecked by capital flight.
Right after the 1992 election, the Wall Street Journal ran a front-page article just informing its readers that there was no reason to fear that
any of the alleged lefties around Clinton would do things differently when they got into office. Of course, the business community already
knew that perfectly well, as you can be by taking a look at the stock market towards the end of the election campaign.
The United States is deeply in debt that was part of the whole Reagan/Bush program, in fact: to put the country so deeply in debt that
there would be virtually no way for the government to pursue programs of social spending anymore. And what being in debt really means
is that the Treasury Department has sold a ton of securities bonds and notes and so on to investors, who then trade them back and
forth on the bond market. About $150 billion worth of U.S. Treasury bonds alone are traded back and forth in a day. If the investing
community which holds those securities doesnt like any U.S. government policies, it can very quickly sell off just a tiny signal amount of
treasury bonds, and that will have the automatic effect of raising the interest rate, which then will have the further automatic effect of raising
the deficit.
There was an article in the London Economist, about the fact that Eastern European countries have been voting Socialists and
Communists back into power. But the basic line of the article was, dont worry about it, because as they said, policy is insulated from
politics meaning, no matter what games these guys play in the political arena, policys going to go on exactly the way it is, because we
control the international currencies, were the only one who can give them loans, we can destroy economies if we want to, they can pretend
they have a democracy if they like anything they please so long as policy remains insulated from politics.
In recent years a completely new form of government is being pioneered, one designed to serve the developing needs of this new
international corporate ruling class its what has sometimes being called an emerging de facto world government. Thats what all of the
new international trade agreements are about; its what the E.E.C is about; its increasingly taking shape in international financial
organisations like the International Monetary Fund, the World Bank, the Inter-American Development Bank, the World Trade Organisation,
The G-7 planning meetings of the rich industrial countries, and so on and so forth.
These are all efforts to try to centralize power in a world economic system geared towards ensuring that the general populations of the
world have no role in decision-making, and the level of policy planning to be so remote from peoples knowledge and understanding that
input that they have no absolutely no idea about the various decisions that are being made that will affect their lives, and certainly couldnt
influence them if they did.
The World Bank has its own term for the phenomenon: they called it technocratic insulation. Technocrats are insulated enough so that
they can have all the democracy they like and can plan their policies without the interference of the masses. In the international business
press, this has been described pretty well as The New Imperial Age.
Excerpted from: Understanding Power by Noam Chomsky.

In order to ensure that the social spending does not increase in the US which one of the following actions was taken?

A)Trade deficit was not allowed to increase.


B)Treasury bonds and notes were sold to investors, who used them to get the government to toe their line.
C)The interest rate was increased leading to the cascading effect of increase in the deficit overnight.
D)No matter who is at the helm of the government, policy would not be affected.
Explanation:-
Option 2. The objective was to keep the US deeply in debt. This was achieved by selling tons of securities to investors. These investors
then controlled the finance. When a policy was not liked by them, they would off load some securities and this would increase the interest
rate and then the deficit. Options 3 and 4, though true, are effects of option 2.

Question No. : 69
DIRECTIONS for the question: The passage given below are followed by a set of question. Choose the most appropriate answer to each
question.
PASSAGE III
Financial speculators dont want growth: what they want is stable currencies, meaning no growth. In fact, the business press talks openly
now about the threat of too much growth, the threat of too much employment: theyre perfectly open about all of this, to one another. And
the reason for it is, people who speculate against currencies are afraid of inflation because it decreases the value of their money, so
therefore its a big threat to them. Currency speculators dont like that, so if they see signs of simulative economic policies or anything that
may bring economic growth, theyll just take their capital out of that companies economy and even a slight withdrawal of that sort can
easily trigger a recession in those countries.
So whats happened as a result of all this is a big drift internationally towards low-growth, low-wage, high-profit economies because
national governments trying to make economic and social policy decisions just have very little leeway to do so by now, or else their
economies will be wrecked by capital flight.
http://www.hitbullseye.com/frmTestView.aspx

37/59

7/7/2015

about:blank

Right after the 1992 election, the Wall Street Journal ran a front-page article just informing its readers that there was no reason to fear that
any of the alleged lefties around Clinton would do things differently when they got into office. Of course, the business community already
knew that perfectly well, as you can be by taking a look at the stock market towards the end of the election campaign.
The United States is deeply in debt that was part of the whole Reagan/Bush program, in fact: to put the country so deeply in debt that
there would be virtually no way for the government to pursue programs of social spending anymore. And what being in debt really means
is that the Treasury Department has sold a ton of securities bonds and notes and so on to investors, who then trade them back and
forth on the bond market. About $150 billion worth of U.S. Treasury bonds alone are traded back and forth in a day. If the investing
community which holds those securities doesnt like any U.S. government policies, it can very quickly sell off just a tiny signal amount of
treasury bonds, and that will have the automatic effect of raising the interest rate, which then will have the further automatic effect of raising
the deficit.
There was an article in the London Economist, about the fact that Eastern European countries have been voting Socialists and
Communists back into power. But the basic line of the article was, dont worry about it, because as they said, policy is insulated from
politics meaning, no matter what games these guys play in the political arena, policys going to go on exactly the way it is, because we
control the international currencies, were the only one who can give them loans, we can destroy economies if we want to, they can pretend
they have a democracy if they like anything they please so long as policy remains insulated from politics.
In recent years a completely new form of government is being pioneered, one designed to serve the developing needs of this new
international corporate ruling class its what has sometimes being called an emerging de facto world government. Thats what all of the
new international trade agreements are about; its what the E.E.C is about; its increasingly taking shape in international financial
organisations like the International Monetary Fund, the World Bank, the Inter-American Development Bank, the World Trade Organisation,
The G-7 planning meetings of the rich industrial countries, and so on and so forth.
These are all efforts to try to centralize power in a world economic system geared towards ensuring that the general populations of the
world have no role in decision-making, and the level of policy planning to be so remote from peoples knowledge and understanding that
input that they have no absolutely no idea about the various decisions that are being made that will affect their lives, and certainly couldnt
influence them if they did.
The World Bank has its own term for the phenomenon: they called it technocratic insulation. Technocrats are insulated enough so that
they can have all the democracy they like and can plan their policies without the interference of the masses. In the international business
press, this has been described pretty well as The New Imperial Age.
Excerpted from: Understanding Power by Noam Chomsky.

The author tries to point out through the passage that

A)The one in office determines the way the policies will be shaped.
B)Growth and increased employment changes the values of the currencies which are appreciated by the investors.
C) Stimulating policies are being brow beaten and economies are languishing because of policies over which the governments have
little control.

D)The US is unaffected by the policies implemented with social development going on in full swing.
Explanation:-
Option 3. The author states that the people in offices are only nominal heads and hence option 1 is incorrect.
The investors do not want inflation and that happens when there is growth and increased employment. Hence 2 is incorrect.
The US is in great debt because policies are not allowed to be implemented the policies would bring about social development and growth
which the investors do not want. Hence option 4 is incorrect.
The author stresses the fact that governments are held to ransom by the policy makers as the policy making has nothing to do with either
the government or the common man.

Question No. : 70
DIRECTIONS for the question: The passage given below are followed by a set of question. Choose the most appropriate answer to each
question.
PASSAGE III
Financial speculators dont want growth: what they want is stable currencies, meaning no growth. In fact, the business press talks openly
now about the threat of too much growth, the threat of too much employment: theyre perfectly open about all of this, to one another. And
the reason for it is, people who speculate against currencies are afraid of inflation because it decreases the value of their money, so
therefore its a big threat to them. Currency speculators dont like that, so if they see signs of simulative economic policies or anything that
may bring economic growth, theyll just take their capital out of that companies economy and even a slight withdrawal of that sort can
easily trigger a recession in those countries.
So whats happened as a result of all this is a big drift internationally towards low-growth, low-wage, high-profit economies because
national governments trying to make economic and social policy decisions just have very little leeway to do so by now, or else their
economies will be wrecked by capital flight.
Right after the 1992 election, the Wall Street Journal ran a front-page article just informing its readers that there was no reason to fear that
any of the alleged lefties around Clinton would do things differently when they got into office. Of course, the business community already
knew that perfectly well, as you can be by taking a look at the stock market towards the end of the election campaign.
The United States is deeply in debt that was part of the whole Reagan/Bush program, in fact: to put the country so deeply in debt that
there would be virtually no way for the government to pursue programs of social spending anymore. And what being in debt really means
is that the Treasury Department has sold a ton of securities bonds and notes and so on to investors, who then trade them back and
forth on the bond market. About $150 billion worth of U.S. Treasury bonds alone are traded back and forth in a day. If the investing
http://www.hitbullseye.com/frmTestView.aspx

38/59

7/7/2015

about:blank

community which holds those securities doesnt like any U.S. government policies, it can very quickly sell off just a tiny signal amount of
treasury bonds, and that will have the automatic effect of raising the interest rate, which then will have the further automatic effect of raising
the deficit.
There was an article in the London Economist, about the fact that Eastern European countries have been voting Socialists and
Communists back into power. But the basic line of the article was, dont worry about it, because as they said, policy is insulated from
politics meaning, no matter what games these guys play in the political arena, policys going to go on exactly the way it is, because we
control the international currencies, were the only one who can give them loans, we can destroy economies if we want to, they can pretend
they have a democracy if they like anything they please so long as policy remains insulated from politics.
In recent years a completely new form of government is being pioneered, one designed to serve the developing needs of this new
international corporate ruling class its what has sometimes being called an emerging de facto world government. Thats what all of the
new international trade agreements are about; its what the E.E.C is about; its increasingly taking shape in international financial
organisations like the International Monetary Fund, the World Bank, the Inter-American Development Bank, the World Trade Organisation,
The G-7 planning meetings of the rich industrial countries, and so on and so forth.
These are all efforts to try to centralize power in a world economic system geared towards ensuring that the general populations of the
world have no role in decision-making, and the level of policy planning to be so remote from peoples knowledge and understanding that
input that they have no absolutely no idea about the various decisions that are being made that will affect their lives, and certainly couldnt
influence them if they did.
The World Bank has its own term for the phenomenon: they called it technocratic insulation. Technocrats are insulated enough so that
they can have all the democracy they like and can plan their policies without the interference of the masses. In the international business
press, this has been described pretty well as The New Imperial Age.
Excerpted from: Understanding Power by Noam Chomsky.

The author agrees with the fact that the world is going towards the New Imperial Age because

A)The people who make the policies are interested in the growth of the nation as a whole.
B)Policy makers formulate policies hand in glove with investors and hence effectively block any interference from the democratic
people.

C)The population has no idea whether the framed polices benefit it nor does it have a say in the actual framing of the policies.
D)Policy makers formulate policies away from the public eye lest the common man might want economic growth instead of profits.
Explanation:-
Option 3. The author time and again stresses that policies cannot be controlled by the government. Moreover the masses neither have
knowledge about the various decisions taken nor can they influence any of the decisions made. The policies are not formulated by the
investors, but the policies that are formulated are only those that benefit the investor. Hence, Option 2 incorrect. The policy makers are
interested in stable currencies and no growth. Hence 1 is incorrect. The option 4 is the truth but that does not show that we are heading
towards the New Imperial Age. (Imperial means actions like merging of bank capital with industrial capital to create finance capital; industry
is increasingly dominated by monopolies; the export of capital becomes more important than the export of commodities)

Question No. : 71
DIRECTIONS for the question: Refer to the following information and answer the question given below.

5 girls, Tanushree, Amisha, Raveena, Preeti and Nandita, are sitting in the first row in a mathematics class. Each girl has a favourite
chocolate bar (Dairy Milk, Perk, Bournville, Kit Kat and Crackle), a favourite colour (green, lilac, pink, blue and purple), a pet (cat,
horse, puppy, rabbit and parrot), a favourite sport (swimming, badminton, horse riding, tennis and hockey), and would like to
holiday at a certain place (Singapore, Maldives, Hong Kong, Canada and Australia). The teacher, sitting in the last row provides
additional information about the positions of the girls and about what they like.

Tanushree likes Bournville.


The girl who owns the cat likes swimming.
Amisha eats Dairy Milk.
Raveena is on the right of Preeti.
Nandita is the first girl on the left.
The first girl on the right likes swimming.
The girl who eats Perk owns a horse.
The girl in the middle eats Dairy Milk.
Raveena likes green.
The girl who wants to go to Singapore sits next to the girl who sits in the middle.
The girl who wants to go to Maldives likes lilac.
The girl who likes Bournville sits next to the girl who wants to go to Hong Kong.
The girl who likes pink wants to go to Hong Kong.
The girl who sits first on the left likes lilac.
The girl who likes blue owns the puppy.
http://www.hitbullseye.com/frmTestView.aspx

39/59

7/7/2015

about:blank

The girl who likes badminton sits next to the girl who has the cat.
The girl on the right of the girl who likes tennis likes horse riding.
The girl next to the girl who likes Perk likes Kit Kat.
The girl who likes purple wants to go to Canada.
The girl who likes Crackle owns the rabbit.
The girl who likes badminton sits next to the girl who plays hockey.
Raveena wants to go to Australia
The girl who likes _______ owns the parrot.
A)lilac

B)purple

C)hockey

D)horse riding

Explanation:-
Suppose the seats are numbered 1 to 5 from left to right. Nandita sits at 1, she likes lilac and wants to go to Maldives.
The person who like swimming and owns the cat sits at 5. Since the girl who owns the cat is at 5, the girl who likes badminton
must be at 4. Since the girl who likes hockey sits next to the girl who likes badminton, we can conclude that Amisha, who eats
Dairy Milk and likes hockey, sits at 3. Raveena is on the right of Preeti. So Raveena, who wants to go to Australia and likes green,
must be at 5 and Preeti must be at 4. Now, Tanushree, who like the Bournville must be at 2 and the girl who wants to go to Hong
Kong and likes pink must be at 3.Since the girl on the right of the girl who likes horse riding likes tennis, Nandita must like tennis
and Tanushree must like horse riding.Since the girls who like Perk and Kit Kat are next to each other, Nandita must like the
Crackle and she owns the rabbit. Since the girl who likes Perk owns a horse, she must be Preeti and Raveena must like Kit Kat. So
the girl who likes blue and owns the puppy must be Tanushree. Since the girl who likes purple wants to go to Canada, she must be
Preeti, and therefore Tanushree wants to go to Singapore and Amisha owns the parrot. The correct order is as follows:
Seat 1: Nandita eats Crackle, likes lilac, owns the rabbit, plays tennis and wants to holiday in Maldives.
Seat 2: Tanushree eats Bournville, likes blue, owns the puppy, likes horse riding and wants to holiday in Singapore.
Seat 3: Amisha eats Dairy Milk, likes pink, owns the parrot, plays hockey and wants to holiday in Hong Kong.
Seat 4: Preeti eats Perk, likes purple, owns the horse, plays badminton and wants to holiday in Canada.
Seat 5: Raveena eats Kit Kat, likes Green, owns the cat, likes swimming and wants to holiday in Australia. The girl who owns the
parrot likes hockey.

Question No. : 72
DIRECTIONS for the question: Refer to the following information and answer the question given below.

5 girls, Tanushree, Amisha, Raveena, Preeti and Nandita, are sitting in the first row in a mathematics class. Each girl has a favourite
chocolate bar (Dairy Milk, Perk, Bournville, Kit Kat and Crackle), a favourite colour (green, lilac, pink, blue and purple), a pet (cat,
horse, puppy, rabbit and parrot), a favourite sport (swimming, badminton, horse riding, tennis and hockey), and would like to
holiday at a certain place (Singapore, Maldives, Hong Kong, Canada and Australia). The teacher, sitting in the last row provides
additional information about the positions of the girls and about what they like.

Tanushree likes Bournville.


The girl who owns the cat likes swimming.
Amisha eats Dairy Milk.
Raveena is on the right of Preeti.
Nandita is the first girl on the left.
The first girl on the right likes swimming.
The girl who eats Perk owns a horse.
The girl in the middle eats Dairy Milk.
Raveena likes green.
The girl who wants to go to Singapore sits next to the girl who sits in
the middle.
The girl who wants to go to Maldives likes lilac.
The girl who likes Bournville sits next to the girl who wants to go to
Hong Kong.
The girl who likes pink wants to go to Hong Kong.
The girl who sits first on the left likes lilac.
The girl who likes blue owns the puppy.
The girl who likes badminton sits next to the girl who has the cat.
The girl on the right of the girl who likes tennis likes horse riding.
The girl next to the girl who likes Perk likes Kit Kat.
The girl who likes purple wants to go to Canada.
The girl who likes Crackle owns the rabbit.
http://www.hitbullseye.com/frmTestView.aspx

40/59

7/7/2015

about:blank

The girl who likes badminton sits next to the girl who plays hockey.
Raveena wants to go to Australia
The girl who wants to holiday in Singapore ___________.
A)likes Bournville

B)likes tennis

C)likes purple

D)owns the cat

Explanation:-
Suppose the seats are numbered 1 to 5 from left to right. Nandita sits at 1, she likes lilac and wants to go to Maldives.
The person who like swimming and owns the cat sits at 5. Since the girl who owns the cat is at 5, the girl who likes badminton
must be at 4. Since the girl who likes hockey sits next to the girl who likes badminton, we can conclude that Amisha, who eats
Dairy Milk and likes hockey, sits at 3. Raveena is on the right of Preeti. So Raveena, who wants to go to Australia and likes green,
must be at 5 and Preeti must be at 4. Now, Tanushree, who like the Bournville must be at 2 and the girl who wants to go to Hong
Kong and likes pink must be at 3.Since the girl on the right of the girl who likes horse riding likes tennis, Nandita must like tennis
and Tanushree must like horse riding. Since the girls who like Perk and Kit Kat are next to each other, Nandita must like the
Crackle and she owns the rabbit. Since the girl who likes Perk owns a horse, she must be Preeti and Raveena must like Kit Kat. So
the girl who likes blue and owns the puppy must be Tanushree. Since the girl who likes purple wants to go to Canada, she must be
Preeti, and therefore Tanushree wants to go to Singapore and Amisha owns the parrot.
The correct order is as follows:
Seat 1: Nandita eats Crackle, likes lilac, owns the rabbit, plays tennis and wants to holiday in Maldives.
Seat 2: Tanushree eats Bournville, likes blue, owns the puppy, likes horse riding and wants to holiday in Singapore.
Seat 3: Amisha eats Dairy Milk, likes pink, owns the parrot, plays hockey and wants to holiday in Hong Kong.
Seat 4: Preeti eats Perk, likes purple, owns the horse, plays badminton and wants to holiday in Canada.
Seat 5: Raveena eats Kit Kat, likes Green, owns the cat, likes swimming and wants to holiday in Australia. The girl who wants to
holiday in Singapore likes Bournville.

Question No. : 73
DIRECTIONS for the question: Refer to the following information and answer the question given below.

5 girls, Tanushree, Amisha, Raveena, Preeti and Nandita, are sitting in the first row in a mathematics class. Each girl has a favourite
chocolate bar (Dairy Milk, Perk, Bournville, Kit Kat and Crackle), a favourite colour (green, lilac, pink, blue and purple), a pet (cat,
horse, puppy, rabbit and parrot), a favourite sport (swimming, badminton, horse riding, tennis and hockey), and would like to
holiday at a certain place (Singapore, Maldives, Hong Kong, Canada and Australia). The teacher, sitting in the last row provides
additional information about the positions of the girls and about what they like.

Tanushree likes Bournville.


The girl who owns the cat likes swimming.
Amisha eats Dairy Milk.
Raveena is on the right of Preeti.
Nandita is the first girl on the left.
The first girl on the right likes swimming.
The girl who eats Perk owns a horse.
The girl in the middle eats Dairy Milk.
Raveena likes green.
The girl who wants to go to Singapore sits next to the girl who sits in
the middle.
The girl who wants to go to Maldives likes lilac.
The girl who likes Bournville sits next to the girl who wants to go to
Hong Kong.
The girl who likes pink wants to go to Hong Kong.
The girl who sits first on the left likes lilac.
The girl who likes blue owns the puppy.
The girl who likes badminton sits next to the girl who has the cat.
The girl on the right of the girl who likes tennis likes horse riding.
The girl next to the girl who likes Perk likes Kit Kat.
The girl who likes purple wants to go to Canada.
The girl who likes Crackle owns the rabbit.
The girl who likes badminton sits next to the girl who plays hockey.
Raveena wants to go to Australia
The girl who likes horse riding sits next to the girl who __________.
A)likes Dairy Milk

B)likes badminton

http://www.hitbullseye.com/frmTestView.aspx

C)likes green

D)owns the horse


41/59

7/7/2015

about:blank

Explanation:-
Suppose the seats are numbered 1 to 5 from left to right. Nandita sits at 1, she likes lilac and wants to go to Maldives.
The person who like swimming and owns the cat sits at 5. Since the girl who owns the cat is at 5, the girl who likes badminton
must be at 4. Since the girl who likes hockey sits next to the girl who likes badminton, we can conclude that Amisha, who eats
Dairy Milk and likes hockey, sits at 3. Raveena is on the right of Preeti. So Raveena, who wants to go to Australia and likes green,
must be at 5 and Preeti must be at 4. Now, Tanushree, who like the Bournville must be at 2 and the girl who wants to go to Hong
Kong and likes pink must be at 3.Since the girl on the right of the girl who likes horse riding likes tennis, Nandita must like tennis
and Tanushree must like horse riding. Since the girls who like Perk and Kit Kat are next to each other, Nandita must like the
Crackle and she owns the rabbit. Since the girl who likes Perk owns a horse, she must be Preeti and Raveena must like Kit Kat. So
the girl who likes blue and owns the puppy must be Tanushree. Since the girl who likes purple wants to go to Canada, she must be
Preeti, and therefore Tanushree wants to go to Singapore and Amisha owns the parrot.
The correct order is as follows:
Seat 1: Nandita eats Crackle, likes lilac, owns the rabbit, plays tennis and wants to holiday in Maldives.
Seat 2: Tanushree eats Bournville, likes blue, owns the puppy, likes horse riding and wants to holiday in Singapore.
Seat 3: Amisha eats Dairy Milk, likes pink, owns the parrot, plays hockey and wants to holiday in Hong Kong.
Seat 4: Preeti eats Perk, likes purple, owns the horse, plays badminton and wants to holiday in Canada.
Seat 5: Raveena eats Kit Kat, likes Green, owns the cat, likes swimming and wants to holiday in Australia. The girl who likes horse
riding sits next to the girl who likes Dairy Milk.

Question No. : 74
DIRECTIONS for the question: Refer to the following information and answer the question given below.

5 girls, Tanushree, Amisha, Raveena, Preeti and Nandita, are sitting in the first row in a mathematics class. Each girl has a favourite
chocolate bar (Dairy Milk, Perk, Bournville, Kit Kat and Crackle), a favourite colour (green, lilac, pink, blue and purple), a pet (cat,
horse, puppy, rabbit and parrot), a favourite sport (swimming, badminton, horse riding, tennis and hockey), and would like to
holiday at a certain place (Singapore, Maldives, Hong Kong, Canada and Australia). The teacher, sitting in the last row provides
additional information about the positions of the girls and about what they like.

Tanushree likes Bournville.


The girl who owns the cat likes swimming.
Amisha eats Dairy Milk.
Raveena is on the right of Preeti.
Nandita is the first girl on the left.
The first girl on the right likes swimming.
The girl who eats Perk owns a horse.
The girl in the middle eats Dairy Milk.
Raveena likes green.
The girl who wants to go to Singapore sits next to the girl who sits in
the middle.
The girl who wants to go to Maldives likes lilac.
The girl who likes Bournville sits next to the girl who wants to go to
Hong Kong.
The girl who likes pink wants to go to Hong Kong.
The girl who sits first on the left likes lilac.
The girl who likes blue owns the puppy.
The girl who likes badminton sits next to the girl who has the cat.
The girl on the right of the girl who likes tennis likes horse riding.
The girl next to the girl who likes Perk likes Kit Kat.
The girl who likes purple wants to go to Canada.
The girl who likes Crackle owns the rabbit.
The girl who likes badminton sits next to the girl who plays hockey.
Raveena wants to go to Australia
Which of the following statements are not true?

I. Amisha owns the parrot. II. Nandita likes Crackle.


III. Preeti owns the puppy. IV. Raveena like Perk.
A)I and II

B)I, II and III

C)III and IV

D)II, III and IV

Explanation:-
Suppose the seats are numbered 1 to 5 from left to right. Nandita sits at 1, she likes lilac and wants to go to Maldives.
The person who like swimming and owns the cat sits at 5. Since the girl who owns the cat is at 5, the girl who likes badminton
http://www.hitbullseye.com/frmTestView.aspx

42/59

7/7/2015

about:blank

must be at 4. Since the girl who likes hockey sits next to the girl who likes badminton, we can conclude that Amisha, who eats
Dairy Milk and likes hockey, sits at 3. Raveena is on the right of Preeti. So Raveena, who wants to go to Australia and likes green,
must be at 5 and Preeti must be at 4. Now, Tanushree, who like the Bournville must be at 2 and the girl who wants to go to Hong
Kong and likes pink must be at 3.Since the girl on the right of the girl who likes horse riding likes tennis, Nandita must like tennis
and Tanushree must like horse riding. Since the girls who like Perk and Kit Kat are next to each other, Nandita must like the
Crackle and she owns the rabbit. Since the girl who likes Perk owns a horse, she must be Preeti and Raveena must like Kit Kat. So
the girl who likes blue and owns the puppy must be Tanushree. Since the girl who likes purple wants to go to Canada, she must be
Preeti, and therefore Tanushree wants to go to Singapore and Amisha owns the parrot.
The correct order is as follows:
Seat 1: Nandita eats Crackle, likes lilac, owns the rabbit, plays tennis and wants to holiday in Maldives.
Seat 2: Tanushree eats Bournville, likes blue, owns the puppy, likes horse riding and wants to holiday in Singapore.
Seat 3: Amisha eats Dairy Milk, likes pink, owns the parrot, plays hockey and wants to holiday in Hong Kong.
Seat 4: Preeti eats Perk, likes purple, owns the horse, plays badminton and wants to holiday in Canada.
Seat 5: Raveena eats Kit Kat, likes Green, owns the cat, likes swimming and wants to holiday in Australia. From the above analysis
we know that Preeti owns the horse and Raveena likes Kit Kat. So statements III and IV are false.
DIRECTIONS for the question: Read the information given below and answer the question that follows.
Question No. : 75
A school prefect has been assigned the task of completing the details of 12 students of the school who are going to participate in
some inter school competitions at the state level. Some information about these students is shown in the table given below. The
students participate in different activities and have been allotted different groups (called houses) for their activities.

It is known that 3 students have been selected from each group.


There are four floors in the school with classes on each floor given as:

First floor: Classes 3rd, 2nd, 1st


Second floor: Classes 6th, 5th, 4th
Third floor: Classes 9th, 8th, 7th
Fourth floor: Classes 12th, 11th, 10th

Student

Group

Sport

Cultural

Floor

TABLE TENNIS

THIRD

JOY

BASKET BALL

SECOND

DEBATE

SECOND

FOOTBALL

THEATRE

FIRST

BASKET BALL

THEATRE

THIRD

DEBATE

FOURTH

JOY

HOCKEY

SECOND

FOOTBALL

CHOIR

FIRST

TABLE TENNIS

THEATRE

THIRD

BASKET BALL

FOURTH

LOVE

HOCKEY

DEBATE

THIRD

PEACE

FOOTBALL

DANCE

FIRST

Additional Information:

a. The two table tennis players have consecutive classes on the third floor. They participate in dance and theatre.
b. Both students from 6th class are in debate group.

c. Another student from eighth is in Joy house and participates in theatre. Maximum participation is from 8th class.
d. All three hockey players are in the debate team which has four members.
e. Students from the first floor do not participate in debate.
f. The 8th class student who plays basketball is in Joy house.

g. Out of the four football players no one has a class on the third or fourth floor. The football player on the second floor is in 6th
class.
h. Two of the basketball players participate in dance, while one is in 4th class the other is in 12th class.
http://www.hitbullseye.com/frmTestView.aspx

43/59

7/7/2015

about:blank

i. There are no students from three consecutive classes.


j. The only student from seventh is in purity house and participates in dance.
k. Two of the hockey players are in love house.
l. The student from peace house who plays football is in 1st class. The other football players are in purity house or love house.
m. The only student for choir is in love house.
Among the following which students are in class 8th?
A)Q, R, W

B)R, T, Q

C)W, Q, T

D)Can't be determined

Explanation:-
According to the information given:

1. The students are from classes 12th, 8th, 7th, 6th, 4th and 1st.
2. According to information (i), the three possible consecutive classes are 9th, 10th and 11th.
3. So the student M on the fourth floor must be from 12th.

4. As there are three students on the first floor, namely, N, U, V and only V has been identified as being in 1st class, it is difficult to
say from which class the students on the first floor are from.

5. According to information b, c, h and above deductions, it can be ascertained that there are two students in 12th, one student in
7th, two students in 6th, one student in 4th, three students on first floor, one of whom is certainly in 1st. The remaining 3 students
are from 8th.
6. As 3 students have been selected from each group, there must be only 4 houses.
7. According to information j, m and l: R, X and N are in purity house.
8. O, T and P are in Joy house; M, W and U are in love house; Q, Y and V are in peace house.
Thus, the table can be completed as:
Condition Student

Group

Sport

Cultural

Class

Floor

a, j

PURITY

Table
Tennis

Dance

Seventh

Third

JOY

Basket
Ball

Dance

Forth

Second

g, l, b

PURITY Football

Debate

Sixth

Second

PURITY Football Theatre

Third

First/Second/Third

JOY

Basket
Ball

Theatre

Eighth

Third

LOVE

Hockey

Debate

Twelfth

Fourth

d, b

JOY

Hockey

Debate

Sixth

Second

LOVE

Football

Choir

Second

First/Second/Third

PEACE

Table
Tennis

Theatre

Eighth

Third

PEACE

Basket
Ball

Dance

Twelfth

Fourth

LOVE

Hockey

Debate

Eighth

Third

PEACE

Football

Dance

First

First

Hence W, Q and T are in class 8th.

DIRECTIONS for the question: Read the information given below and answer the question that follows.
Question No. : 76
A school prefect has been assigned the task of completing the details of 12 students of the school who are going to participate in
some inter school competitions at the state level. Some information about these students is shown in the table given below. The
students participate in different activities and have been allotted different groups (called houses) for their activities.

It is known that 3 students have been selected from each group.


There are four floors in the school with classes on each floor given as:

First floor: Classes 3rd, 2nd, 1st


http://www.hitbullseye.com/frmTestView.aspx

44/59

7/7/2015

about:blank

Second floor: Classes 6th, 5th, 4th

Third floor: Classes 9th, 8th, 7th


Fourth floor: Classes 12th, 11th, 10th

Student

Group

Sport

Cultural

Floor

TABLE TENNIS

THIRD

JOY

BASKET BALL

SECOND

DEBATE

SECOND

FOOTBALL

THEATRE

FIRST

BASKET BALL

THEATRE

THIRD

DEBATE

FOURTH

JOY

HOCKEY

SECOND

FOOTBALL

CHOIR

FIRST

TABLE TENNIS

THEATRE

THIRD

BASKET BALL

FOURTH

LOVE

HOCKEY

DEBATE

THIRD

PEACE

FOOTBALL

DANCE

FIRST

Additional Information:

a. The two table tennis players have consecutive classes on the third floor. They participate in dance and theatre.
b. Both students from 6th class are in debate group.
c. Another student from eighth is in Joy house and participates in theatre. Maximum participation is from 8th class.
d. All three hockey players are in the debate team which has four members.
e. Students from the first floor do not participate in debate.
f. The 8th class student who plays basketball is in Joy house.
g. Out of the four football players no one has a class on the third or fourth floor. The football player on the second floor is in 6th
class.
h. Two of the basketball players participate in dance, while one is in 4th class the other is in 12th class.
i. There are no students from three consecutive classes.
j. The only student from seventh is in purity house and participates in dance.
k. Two of the hockey players are in love house.
l. The student from peace house who plays football is in 1st class. The other football players are in purity house or love house.
m. The only student for choir is in love house.
Which are the three consecutive classes from which there are no participating students?
A)2nd, 3rd, 4th

B)4th, 5th, 6th

C)7th, 8th, 9th

D)9th, 10th, 11th

Explanation:-
According to the information given:

1. The students are from classes 12th, 8th, 7th, 6th, 4th and 1st.
2. According to information (i), the three possible consecutive classes are 9th, 10th and 11th.
3. So the student M on the fourth floor must be from 12th.
4. As there are three students on the first floor, namely, N, U, V and only V has been identified as being in 1st class, it is difficult to
say from which class the students on the first floor are from.
5. According to information b, c, h and above deductions, it can be ascertained that there are two students in 12th, one student in
7th, two students in 6th, one student in 4th, three students on first floor, one of whom is certainly in 1st. The remaining 3 students
are from 8th.
6. As 3 students have been selected from each group, there must be only 4 houses.
7. According to information j, m and l: R, X and N are in purity house.
8. O, T and P are in Joy house; M, W and U are in love house; Q, Y and V are in peace house.
Thus, the table can be completed as:
Condition Student
a, j

http://www.hitbullseye.com/frmTestView.aspx

Group

Sport

PURITY Table Tennis


JOY

Basket Ball

Cultural

Class

Floor

Dance

Seventh

Third

Dance

Forth

Second
45/59

7/7/2015

about:blank

g, l, b

PURITY

Football

Debate

Sixth

Second

PURITY

Football

Theatre

Third

First/Second/Third

JOY

Basket Ball

Theatre

Eighth

Third

LOVE

Hockey

Debate

Twelfth

Fourth

d, b

JOY

Hockey

Debate

Sixth

Second

LOVE

Football

Choir

Second

First/Second/Third

PEACE

Eighth

Third

PEACE

Basket Ball

Dance

Twelfth

Fourth

LOVE

Hockey

Debate

Eighth

Third

PEACE

Football

Dance

First

First

Table Tennis Theatre

Hence students from 9th, 10th and 11th did not participate.

DIRECTIONS for the question: Read the information given below and answer the question that follows.
Question No. : 77
A school prefect has been assigned the task of completing the details of 12 students of the school who are going to participate in
some inter school competitions at the state level. Some information about these students is shown in the table given below. The
students participate in different activities and have been allotted different groups (called houses) for their activities.

It is known that 3 students have been selected from each group.


There are four floors in the school with classes on each floor given as:

First floor: Classes 3rd, 2nd, 1st


Second floor: Classes 6th, 5th, 4th
Third floor: Classes 9th, 8th, 7th
Fourth floor: Classes 12th, 11th, 10th

Student

Group

Sport

Cultural

Floor

TABLE TENNIS

THIRD

JOY

BASKET BALL

SECOND

DEBATE

SECOND

FOOTBALL

THEATRE

FIRST

BASKET BALL

THEATRE

THIRD

DEBATE

FOURTH

JOY

HOCKEY

SECOND

FOOTBALL

CHOIR

FIRST

TABLE TENNIS

THEATRE

THIRD

BASKET BALL

FOURTH

LOVE

HOCKEY

DEBATE

THIRD

PEACE

FOOTBALL

DANCE

FIRST

Additional Information:

a. The two table tennis players have consecutive classes on the third floor. They participate in dance and theatre.
b. Both students from 6th class are in debate group.
c. Another student from eighth is in Joy house and participates in theatre. Maximum participation is from 8th class.
d. All three hockey players are in the debate team which has four members.
e. Students from the first floor do not participate in debate.
f. The 8th class student who plays basketball is in Joy house.
g. Out of the four football players no one has a class on the third or fourth floor. The football player on the second floor is in 6th
class.
http://www.hitbullseye.com/frmTestView.aspx

46/59

7/7/2015

about:blank

h. Two of the basketball players participate in dance, while one is in 4th class the other is in 12th class.
i. There are no students from three consecutive classes.
j. The only student from seventh is in purity house and participates in dance.
k. Two of the hockey players are in love house.
l. The student from peace house who plays football is in 1st class. The other football players are in purity house or love house.
m. The only student for choir is in love house.
Student Y is in which house and participates in which cultural activity?
A)Love, dance

B)Joy, theatre

C)Purity, theatre

D)Peace, dance

Explanation:-
According to the information given:
1. The students are from classes 12th, 8th, 7th, 6th, 4th and 1st.
2. According to information (i), the three possible consecutive classes are 9th, 10th and 11th.

3. So the student M on the fourth floor must be from 12th.


4. As there are three students on the first floor, namely, N, U, V and only V has been identified as being in 1st class, it is difficult to
say from which class the students on the first floor are from.
5. According to information b, c, h and above deductions, it can be ascertained that there are two students in 12th, one student in
7th, two students in 6th, one student in 4th, three students on first floor, one of whom is certainly in 1st. The remaining 3 students
are from 8th.
6. As 3 students have been selected from each group, there must be only 4 houses.
7. According to information j, m and l: R, X and N are in purity house.
8. O, T and P are in Joy house; M, W and U are in love house; Q, Y and V are in peace house.
Thus, the table can be completed as:
Condition Student

Group

Sport

PURITY Table Tennis

Cultural

Class

Floor

Dance

Seventh

Third

a, j

JOY

Basket Ball

Dance

Forth

Second

g, l, b

PURITY

Football

Debate

Sixth

Second

PURITY

Football

Theatre

Third

First/Second/Third

JOY

Basket Ball

Theatre

Eighth

Third

LOVE

Hockey

Debate

Twelfth

Fourth

d, b

JOY

Hockey

Debate

Sixth

Second

LOVE

Football

Choir

Second

First/Second/Third

PEACE

Eighth

Third

PEACE

Basket Ball

Dance

Twelfth

Fourth

LOVE

Hockey

Debate

Eighth

Third

PEACE

Football

Dance

First

First

Table Tennis Theatre

Student Y is from Peace house and participates in Dance.

DIRECTIONS for the question: Read the information given below and answer the question that follows.
Question No. : 78
A school prefect has been assigned the task of completing the details of 12 students of the school who are going to participate in
some inter school competitions at the state level. Some information about these students is shown in the table given below. The
students participate in different activities and have been allotted different groups (called houses) for their activities.

It is known that 3 students have been selected from each group.


There are four floors in the school with classes on each floor given as:

First floor: Classes 3rd, 2nd, 1st


Second floor: Classes 6th, 5th, 4th
Third floor: Classes 9th, 8th, 7th
Fourth floor: Classes 12th, 11th, 10th

http://www.hitbullseye.com/frmTestView.aspx

47/59

7/7/2015

about:blank

Student

Group

Sport

Cultural

Floor

TABLE TENNIS

THIRD

JOY

BASKET BALL

SECOND

DEBATE

SECOND

FOOTBALL

THEATRE

FIRST

BASKET BALL

THEATRE

THIRD

DEBATE

FOURTH

JOY

HOCKEY

SECOND

FOOTBALL

CHOIR

FIRST

TABLE TENNIS

THEATRE

THIRD

BASKET BALL

FOURTH

LOVE

HOCKEY

DEBATE

THIRD

PEACE

FOOTBALL

DANCE

FIRST

Additional Information:

a. The two table tennis players have consecutive classes on the third floor. They participate in dance and theatre.
b. Both students from 6th class are in debate group.
c. Another student from eighth is in Joy house and participates in theatre. Maximum participation is from 8th class.
d. All three hockey players are in the debate team which has four members.
e. Students from the first floor do not participate in debate.
f. The 8th class student who plays basketball is in Joy house.
g. Out of the four football players no one has a class on the third or fourth floor. The football player on the second floor is in 6th
class.
h. Two of the basketball players participate in dance, while one is in 4th class the other is in 12th class.
i. There are no students from three consecutive classes.
j. The only student from seventh is in purity house and participates in dance.
k. Two of the hockey players are in love house.
l. The student from peace house who plays football is in 1st class. The other football players are in purity house or love house.
m. The only student for choir is in love house.
Which of the following students are in peace house?
A)R, X, V

B)Q, Y, V

C)M, Y, V

D)N, T, V

Explanation:-
According to the information given:
1. The students are from classes 12th, 8th, 7th, 6th, 4th and 1st.
2. According to information (i), the three possible consecutive classes are 9th, 10th and 11th.
3. So the student M on the fourth floor must be from 12th.

4. As there are three students on the first floor, namely, N, U, V and only V has been identified as being in 1st class, it is difficult to
say from which class the students on the first floor are from.

5. According to information b, c, h and above deductions, it can be ascertained that there are two students in 12th, one student in
7th, two students in 6th, one student in 4th, three students on first floor, one of whom is certainly in 1st. The remaining 3 students
are from 8th.
6. As 3 students have been selected from each group, there must be only 4 houses.
7. According to information j, m and l: R, X and N are in purity house.
8. O, T and P are in Joy house; M, W and U are in love house; Q, Y and V are in peace house.
Thus, the table can be completed as:
Condition Student

Group

Sport

Class

Floor

Dance

Seventh

Third

a, j

JOY

Basket Ball

Dance

Forth

Second

g, l, b

PURITY

Football

Debate

Sixth

Second

PURITY

Football

Theatre

Third

First/Second/Third

JOY

Basket Ball

Theatre

Eighth

Third

LOVE

Hockey

Debate

Twelfth

Fourth

http://www.hitbullseye.com/frmTestView.aspx

PURITY Table Tennis

Cultural

48/59

7/7/2015

about:blank

d, b

JOY

Hockey

Debate

Sixth

Second

LOVE

Football

Choir

Second

First/Second/Third

PEACE

Eighth

Third

PEACE

Basket Ball

Dance

Twelfth

Fourth

LOVE

Hockey

Debate

Eighth

Third

PEACE

Football

Dance

First

First

Table Tennis Theatre

Hence Q, Y and V are in peace house.

DIRECTIONS for the question: Refer the following and answer the question that follows:

If highways were restricted to cars and only those trucks with capacity of less than 8 tons, most of the truck traffic would be forced
to run outside highways. Such a reduction in the amount of truck traffic would reduce the risk of collisions on highways.
Question No. : 79

The conclusion drawn in the first sentence depends on which of the following assumptions?
A)The roads outside highway would be as convenient as highway for most drivers of trucks..
B)Most roads outside highways are not ready to handle truck traffic.
C)Most trucks that are currently running in highway have a capacity of more than 8 tons.
D)Cars are at greater risk of becoming involved in collisions than are trucks.
Explanation:-
The first sentence concludes that prohibiting trucks with capacity of more than 8 tons from highway would force most trucks away
from highways.
This conclusion cannot be true unless it is true that, as 3 says, most trucks that use highways have capacity of more than 8 tons.
Therefore, the first sentence's conclusion assumes this choice, which is thus the best answer.

DIRECTIONS for the question: Refer the following and answer the question that follows:
If highways were restricted to cars and only those trucks with capacity of less than 8 tons, most of the truck traffic would be forced to run
outside highways. Such a reduction in the amount of truck traffic would reduce the risk of collisions on highways.

Question No. : 80

Which of the following, if true, would most strengthen the conclusion drawn in the second sentence?

A)Highways are experiencing overcrowded traffic primarily because of sharp increases in car traffic
B)Many drivers of trucks would rather buy trucks with a capacity of less than 8 tons than be excluded from highways.
C)The number of collisions that occur near highways has decreased in recent years
D)Trucks that have a capacity of more than 8 tons cause a disproportionately large number of collisions on highways
Explanation:-
The second sentence concludes that the reduction described in the first sentence would reduce the risk of collisions in highways.
According to 4, such a reduction would remove precisely the kind of truck that causes a disproportionate number of collisions. Thus 4 is
the best answer

DIRECTIONS for the question: Pick the best option which completes the sentence in the most meaningful manner.
Question No. : 81
The _________ in many parts of the metros has made the _________ of infectious diseases more swift, because pathogens spread
fast in close quarters.
A)squalor circulation B)congestiondeterioration
D)over-development proliferation

C)overcrowdingpropagation

Explanation:-
The words that help us to identify the correct option are close quarters. We are talking about infectious diseases and their spread
http://www.hitbullseye.com/frmTestView.aspx

49/59

7/7/2015

about:blank

would be fast because of people being in close contact with one another. Hence overcrowding or congestion can fit the first blank.
However the statement states that infectious diseases spread fast in close quarters hence the word deterioration cannot fit the
second blank. Propagation is the correct choice for the second blank. Squalor means filth. There is no doubt that filth and dirt will
result in a fast spread of the disease but we are interested in finding a word which talks about infectious diseases spreading from
one person to another.
DIRECTIONS for the question: In the sentence given below one word has been used in different ways. Select the one in which the
usage is incorrect.
Question No. : 82
PHASE
A)Until the matter is gone into at length it will be impossible to discuss any phase of it with exactness
B)On Monday, the Directorate of Education released a list of 11 schools set to be phased out and a charter school
recommended for non-renewal
C)China will phase in planned changes to its loan-loss provisioning rules to give banks time to adapt
D)The current lack of good quality programming doesn't phase the leaders of this up-and-coming company
Explanation:-
As averb,phasemeans to plan or carryout systematically. Its usually followed by in or out.Fazemeansto disrupt the composure
of. So, for example, when you implement a plan little by little, you phase it in. If you are not bothered by something, you are
unfazed.
DIRECTIONS for the question: Given below are five sentences. Identify the sentence(s) that is/are incorrect in terms of grammar
and usage (including spelling, punctuation and logical consistency). Then, choose the most appropriate option.
Question No. : 83
A. She looked at me with tears falling down from her eyes.
B. Many of these children grow in an atmosphere of violence.
C. The question is whether doctors should lengthen life when there is no hope of recovery.
D. Suddenly a wonderful smile lighted up her face.
E. In todays material society, most people think only about money.
A)A, B and C

B)C and E only

C)A, D and E

D)All of them

Explanation:-
In A, tears falling and not tears falling down. In B, grow up instead of grow in (Grow develop or get bigger as part of a natural
process. Grow up pass from childhood into maturity.) In C, instead of lengthen it should be prolong. Lengthen making
something longer, become longer. Prolong prevent a feeling, activity or life from ending. In D, lit instead of lighted. In British
English, the usual past tense and the past participle form of light is lit. Lighted is mainly used before a noun lighted match,
lighted cigarette. In E, the word needed is an adjective. Material is a noun. Materialistic instead of material.

DIRECTIONS for the question : Read the passage and answer the question based on it.
Question No. : 84
In most peoples minds, yawning that slow, exaggerated mouth opening with the long, deep inhalation of air, followed by a
briefer exhalation is the most obvious sign of sleepiness. It is a common behaviour shared by many animals, including not only
our pet dogs and cats but also, crocodiles, snakes, birds, and even some fish. It is certainly true that sleepy people tend to yawn
more than wide
awake people. It is also true that people who say they are bored by what is happening at the moment will tend to
yawn more frequently. However, whether yawning is a sign that you are getting ready for sleep or that you are successfully
fighting off sleep is not known.

Unfortunately, yawns dont just indicate sleepiness. In some animals, yawning is a sign of stress. When a dog-trainer sees a dog
yawning in a dog obedience class, it is usually a sign that the animal is under a good deal of pressure. Perhaps the handler is
pushing too hard or moving too fast for the dog to feel in control of the situation. A moment or two of play and then turning to
another activity is usually enough to banish yawning for quite a while.

Yawning can also be a sign of stress in humans. Once, when observing airborne troops about to take their first parachute jump, I
http://www.hitbullseye.com/frmTestView.aspx

50/59

7/7/2015

about:blank

noticed that several of the soldiers were sitting in the plane and yawning. It was 10 A.M., just after a coffee break, and I doubted
that they were tired; I knew for a fact that they were far too nervous to be bored. When I asked about this, the officer in charge
laughed and said it
was really quite a common behaviour, especially on the first jump.

There is also a social aspect to yawning. Psychologists have placed actors in crowded rooms and auditoriums and had them
deliberately yawn. Within moments, there is usually an increase in yawning by everyone else in the room. Similarly, people who
watch films or videos of others yawning are more likely to yawn. Even just reading about yawning tends to stimulate people to
yawn.

The truth of the matter is that we really dont know what purpose yawning serves. Scientists originally thought that the purpose of
yawning was to increase the amount of oxygen in the blood or to release some accumulated carbon dioxide. We now know that
this is not true, since increasing the concentration of carbon dioxide in the air seems not to increase the likelihood of yawning.

Since yawning seems to be associated with a lot more than the need for sleep, we obviously have to find some other measure of
sleepiness. Some researchers have simply tried to ask people how sleepy they feel at any time using some sort of self
rating scale.
There are, however, problems with getting people to make these types of judgments. Sometimes people simply lie to the
researchers when asked about how sleepy they are. This occurs because in many areas of society admitting that one is fatigued
and sleepy is considered a mark of weakness or lack of ambition and drive. In other instances, people may admit they need four
cups of coffee to make it through the morning, but it may never occur to them that this might be due to the fact that they are so
sleepy that they need stimulation from caffeine to be able to do their required tasks. For these reasons, many researchers have
developed an alternate method to determine how sleepy a person is. It is based upon a simple definition of sleep need: The greater
your sleep need, or the sleepier you are, the faster you will fall asleep if given the opportunity to do so.

What is the primary purpose of the passage?


A)To refute the claim that yawning increases the amount of oxygen in the blood
B)To challenge the assertion that yawning indicates stress and boredom
C)To determine whether yawning is an effective measure of sleepiness
D)To illustrate that yawns are signs of something more than just sleepiness
Explanation:-
The author starts the passage saying that most people feel that yawning is the most obvious example of sleepiness. Then he goes
on to say that boredom and stress also induce yawns. In the last paragraph, he states that since yawning is associated with a lot
more things than denoting sleep, we need to find some other measure of sleepiness. He also concludes the passage stating that
some researchers have developed an alternate method of determining how sleepy a person is. Though option is true it is not the
primary purpose of the passage. Option 2 is incorrect because the author states that yawning indicates much more than
sleepiness, it indicates boredom and stress too.
DIRECTIONS for the question : Read the passage and answer the question based on it.
Question No. : 85
In most peoples minds, yawning that slow, exaggerated mouth opening with the long, deep inhalation of air, followed by a
briefer exhalation is the most obvious sign of sleepiness. It is a common behaviour shared by many animals, including not only
our pet dogs and cats but also, crocodiles, snakes, birds, and even some fish. It is certainly true that sleepy people tend to yawn
more than wide
awake people. It is also true that people who say they are bored by what is happening at the moment will tend to
yawn more frequently. However, whether yawning is a sign that you are getting ready for sleep or that you are successfully
fighting off sleep is not known.

Unfortunately, yawns dont just indicate sleepiness. In some animals, yawning is a sign of stress. When a dog-trainer sees a dog
yawning in a dog obedience class, it is usually a sign that the animal is under a good deal of pressure. Perhaps the handler is
pushing too hard or moving too fast for the dog to feel in control of the situation. A moment or two of play and then turning to
another activity is usually enough to banish yawning for quite a while.

Yawning can also be a sign of stress in humans. Once, when observing airborne troops about to take their first parachute jump, I
noticed that several of the soldiers were sitting in the plane and yawning. It was 10 A.M., just after a coffee break, and I doubted
that they were tired; I knew for a fact that they were far too nervous to be bored. When I asked about this, the officer in charge
laughed and said it
was really quite a common behaviour, especially on the first jump.

There is also a social aspect to yawning. Psychologists have placed actors in crowded rooms and auditoriums and had them
deliberately yawn. Within moments, there is usually an increase in yawning by everyone else in the room. Similarly, people who
http://www.hitbullseye.com/frmTestView.aspx

51/59

7/7/2015

about:blank

watch films or videos of others yawning are more likely to yawn. Even just reading about yawning tends to stimulate people to
yawn.

The truth of the matter is that we really dont know what purpose yawning serves. Scientists originally thought that the purpose of
yawning was to increase the amount of oxygen in the blood or to release some accumulated carbon dioxide. We now know that
this is not true, since increasing the concentration of carbon dioxide in the air seems not to increase the likelihood of yawning.

Since yawning seems to be associated with a lot more than the need for sleep, we obviously have to find some other measure of
sleepiness. Some researchers have simply tried to ask people how sleepy they feel at any time using some sort of self
rating scale.
There are, however, problems with getting people to make these types of judgments. Sometimes people simply lie to the
researchers when asked about how sleepy they are. This occurs because in many areas of society admitting that one is fatigued
and sleepy is considered a mark of weakness or lack of ambition and drive. In other instances, people may admit they need four
cups of coffee to make it through the morning, but it may never occur to them that this might be due to the fact that they are so
sleepy that they need stimulation from caffeine to be able to do their required tasks. For these reasons, many researchers have
developed an alternate method to determine how sleepy a person is. It is based upon a simple definition of sleep need: The greater
your sleep need, or the sleepier you are, the faster you will fall asleep if given the opportunity to do so.
At the beginning of the last paragraph the author tries to
A)Deviate from the topic under discussion B)Evaluate the findings in the earlier paragraphs
C)Accepts a drawback to an approach mentioned in the previous paragraph
D)Brings the discussion back to a problem mentioned earlier in the passage
Explanation:-
The author is not evaluating the earlier paragraphs. He is in a way summarising the findings and then taking the discussion
forward. Hence option 2 is incorrect. Option 1 is incorrect as the author is continuing on the topic of yawning. Drawback is stated
and concluded in second last paragraph itself therefore option 3 is also rejected.
DIRECTIONS for the question : Read the passage and answer the question based on it.
Question No. : 86
In most peoples minds, yawning that slow, exaggerated mouth opening with the long, deep inhalation of air, followed by a
briefer exhalation is the most obvious sign of sleepiness. It is a common behaviour shared by many animals, including not only
our pet dogs and cats but also, crocodiles, snakes, birds, and even some fish. It is certainly true that sleepy people tend to yawn
more than wide
awake people. It is also true that people who say they are bored by what is happening at the moment will tend to
yawn more frequently. However, whether yawning is a sign that you are getting ready for sleep or that you are successfully
fighting off sleep is not known.

Unfortunately, yawns dont just indicate sleepiness. In some animals, yawning is a sign of stress. When a dog-trainer sees a dog
yawning in a dog obedience class, it is usually a sign that the animal is under a good deal of pressure. Perhaps the handler is
pushing too hard or moving too fast for the dog to feel in control of the situation. A moment or two of play and then turning to
another activity is usually enough to banish yawning for quite a while.

Yawning can also be a sign of stress in humans. Once, when observing airborne troops about to take their first parachute jump, I
noticed that several of the soldiers were sitting in the plane and yawning. It was 10 A.M., just after a coffee break, and I doubted
that they were tired; I knew for a fact that they were far too nervous to be bored. When I asked about this, the officer in charge
laughed and said it was really quite a common behaviour, especially on the first jump.

There is also a social aspect to yawning. Psychologists have placed actors in crowded rooms and auditoriums and had them
deliberately yawn. Within moments, there is usually an increase in yawning by everyone else in the room. Similarly, people who
watch films or videos of others yawning are more likely to yawn. Even just reading about yawning tends to stimulate people to
yawn.

The truth of the matter is that we really dont know what purpose yawning serves. Scientists originally thought that the purpose of
yawning was to increase the amount of oxygen in the blood or to release some accumulated carbon dioxide. We now know that
this is not true, since increasing the concentration of carbon dioxide in the air seems not to increase the likelihood of yawning.

Since yawning seems to be associated with a lot more than the need for sleep, we obviously have to find some other measure of
sleepiness. Some researchers have simply tried to ask people how sleepy they feel at any time using some sort of self
rating scale.
There are, however, problems with getting people to make these types of judgments. Sometimes people simply lie to the
researchers when asked about how sleepy they are. This occurs because in many areas of society admitting that one is fatigued
and sleepy is considered a mark of weakness or lack of ambition and drive. In other instances, people may admit they need four
cups of coffee to make it through the morning, but it may never occur to them that this might be due to the fact that they are so
http://www.hitbullseye.com/frmTestView.aspx

52/59

7/7/2015

about:blank

sleepy that they need stimulation from caffeine to be able to do their required tasks. For these reasons, many researchers have
developed an alternate method to determine how sleepy a person is. It is based upon a simple definition of sleep need: The greater
your sleep need, or the sleepier you are, the faster you will fall asleep if given the opportunity to do so.
The author uses all of the following to put forth his views, except
A)An anecdote

B)Illustration

C)Comparison

D)Understatement

Explanation:-
Anecdote : A short and amusing or interesting story about a real incident or person.The anecdote is the incident of the soldiers
who are to take their first parachute jump. Illustration is material used to clarify or explain dog training and people in crowded
places. (dogs yawn when stressed. People yawn in crowded places because of the social aspect the power of suggestion).
Comparison is between animals and human beings both yawn. So it is clear that he is not making an understatement by trying
to find out the reasons behind 'yawning' from different perspectives.
DIRECTIONS for the question: Read the information given below and answer the question that follows.
Question No. : 87
Seven boys are made to stand in a row in order of their increasing heights (shortest boy first). Their names are Arjun, Balvinder,
Chaman, Diljeet, Deepak, Farhaan, Gopal (not in that order). Following statements give information about their positions in the
row.

1. Balvinder is the tallest boy.


2. Farhaan is exactly between Arjun and Diljeet.
3. There are exactly three boys between Diljeet and Gopal.
4. Chaman comes right after Balvinder in height.

According to the information given, how many arrangements are possible?


A)1

B)2

C)3

D)4

Explanation:-
Considering the given statement is following order , we get, From (1)Balvinder is at 7th place. From (4), Diljeet is at 6th place. From
(3) Diljeet and Gopal must be standing at 1st and 5th places. From (2), Arjun, Farhaan and Diljeet are standing at 3rd ,4th and 5th
places respectively or at 3rd,2nd and 1st places respectively. Thus, we get two possible arrangements as:

Shortest to Tallest

Dt

Dk

Dk

Dt

So its clear that two arrangements are possible.

DIRECTIONS for the question: Read the information given below and answer the question that follows.
Question No. : 88
Seven boys are made to stand in a row in order of their increasing heights (shortest boy first). Their names are Arjun, Balvinder,
Chaman, Diljeet, Deepak, Farhaan, Gopal (not in that order). Following statements give information about their positions in the
row.

1. Balvinder is the tallest boy.


2. Farhaan is exactly between Arjun and Diljeet.
3. There are exactly three boys between Diljeet and Gopal.
4. Chaman comes right after Balvinder in height.

How many boys take the same position in the row in all the possible arrangements?
A)2

B)3

C)4

D)5

http://www.hitbullseye.com/frmTestView.aspx

53/59

7/7/2015

about:blank

Explanation:-

Considering the given statement is following order , we get, From (1)Balvinder is at 7th place. From (4), Diljeet is at 6th place. From
(3) Diljeet and Gopal must be standing at 1st and 5th places. From (2), Arjun, Farhaan and Diljeet are standing at 3rd ,4th and 5th
places respectively or at 3rd,2nd and 1st places respectively. Thus, we get two possible arrangements as:

Shortest to Tallest

Dt

Dk

Dk

Dt

3 boys, Arjun, Chaman and Balwinder, occupy the same positions in both arrangements.

DIRECTIONS for the question: Read the information given below and answer the question that follows.
Question No. : 89
Seven boys are made to stand in a row in order of their increasing heights (shortest boy first). Their names are Arjun, Balvinder,
Chaman, Diljeet, Deepak, Farhaan, Gopal (not in that order). Following statements give information about their positions in the
row.

1. Balvinder is the tallest boy.


2. Farhaan is exactly between Arjun and Diljeet.
3. There are exactly three boys between Diljeet and Gopal.
4. Chaman comes right after Balvinder in height.

If it is given that Farhaan is taller than Deepak, then who is the shortest boy?

A)Chaman

B)Deepak

C)Gopal

D)Cannot be determined

Explanation:-
Considering the given statement is following order , we get, From (1)Balvinder is at 7th place. From (4), Diljeet is at 6th place. From
(3) Diljeet and Gopal must be standing at 1st and 5th places. From (2), Arjun, Farhaan and Diljeet are standing at 3rd ,4th and 5th
places respectively or at 3rd,2nd and 1st places respectively. Thus, we get two possible arrangements as:

Shortest to Tallest

Dt

Dk

Dk

Dt

If Farhan is taller than Deepak, we need to refer to the 2nd arrangement, in which Gopal is the shortest boy.

DIRECTIONS for the question: The question consists of five statements labelled A, B, C, D and E which when logically ordered
form a coherent passage. Choose the option that represents the most logical order.
Question No. : 90
A. Prokaryotic sex, on the other hand, is infrequent and inefficient.
B. The creative process of natural selection works by preserving favorable genetic variants from an extensive pool.
C. Major evolutionary change cannot occur unless organisms maintain a large store of genetic variability.
D. Sex can provide variation on this scale, but efficient sexual reproduction requires the packaging of genetic material into discrete
units.
E. Thus, in eukaryotes, sex cells have half the chromosomes of normal body cells and when two cells join to produce an offspring,
the original amount of genetic material is restored.
A)BCDEA

B)CBDAE

C)CBDEA

http://www.hitbullseye.com/frmTestView.aspx

D)BDCEA

54/59

7/7/2015

A)BCDEA

about:blank

B)CBDAE

C)CBDEA

D)BDCEA

Explanation:-
The paragraph talks about evolutionary change, organisms and genetic material. C introduces the idea first. A will follow E, as first
we talk of eukaryotes and then on the other hand on prokaryotic sex. The this in D refers to the idea mentioned in B.
DIRECTIONS for the question: Pick the best option which completes the sentence in the most meaningful manner.
Question No. : 91
The student found writing to be very __________, since she was not terribly ______________ and always struggled to find enough to
say to fill her term papers.
A)morosemawkish

B)perfunctory.. laconic

C)stanchion verbose

D)onerous loquacious

Explanation:-
Onerous means difficult or laborious. Loquacious means verbose or talkative or wordy. If she is struggling to find something to
say, we know we are looking for a word like difficult to describe her attitude towards writing. The information that she was not
terribly tells us that we are looking for something that means the opposite of the rest of the sentence - so a word like good at
writing. Onerous and loquacious both fit in these context. Morose means gloomy. Mawkish means sickening or unpleasant and
inappropriate. A stanchion is a bar used to keep cattle in their cages.
DIRECTIONS for the question: In the given paragraph, the last line has been deleted. Choose the option that logically follows the
paragraph.
Question No. : 92
When any new substitutive technology enters the market for the first time, it is priced higher than the technology it is supposed to
be substituting. This is what happened when CDs were launched. They were priced significantly higher than vinyl records. The
justification was that low volumes increased production costs. With economies of scale and learning curves, the price of CDs did
come down. Yet vinyl records, which had been in existence for a very long period of time, started going up in price to become
almost as expensive as CDs.

This can be explained by the fact that ____

A)Listeners appreciated the enhanced quality of sound in CDs so much that they were ready to pay more for CDs
B)Price sensitive consumers refused to upgrade to CDs
C) As the switch to CDs started happening, the volumes in the vinyl record market reduced, thereby increasing production

costs
D)The requirement to buy a separate CD player was a reason for consumers to continue buying vinyl records
Explanation:-
Just as the low volumes made the cost of CDs high initially, so too when the demand for the vinyl records fell, the cost of the vinyl
records increased.

DIRECTIONS for the question: Given below are five sentences. Identify the sentence(s) that is/are incorrect in terms of grammar
and usage (including spelling, punctuation and logical consistency). Then, choose the most appropriate option.
Question No. : 93
A. Conditions for journalism have never been better; robust media profits, strong legal protections, and sophisticated technology.
B. Yet there is an influential movement, representing the concensus of the professions elite,
C. dedicating to convincing us that all is not well.
D. The book The Elements of Journalism suggests that unless a certain theory of news
E. is adhered to, the country might be annihilated.

A)B and E only

B)D only

C)A, B and C

D)D and E only

Explanation:-
In A, the semicolon needs to be replaced by a colon. In B, the correct spelling is consensus and not concensus. In C, we need a verb
http://www.hitbullseye.com/frmTestView.aspx

55/59

7/7/2015

about:blank

and not a gerund to complete the sentence dedicated instead of dedicating.


DIRECTIONS for the question : Read the passage and answer the question based on it.
Question No. : 94
If the new art is not accessible to everyone, which certainly seems to be the case, this implies that its impulses are not of a
generically human kind. It is an art not for people in general but for a special class who may not be better but who are evidently
different.

Before we go further, one point must be clarified. What is it that the majority of people call aesthetic pleasure? What happens in
their minds when they like a work of art; for example, a play? The answer is easy. They like a play when they become interested
in the human destinies that are represented, when the love and hatred, the joys and sorrows of the dramatic personages so move
them that they participate in it all as though it were happening in real life. And they call a work good if it succeeds in creating the
illusion necessary to make the imaginary personages appear like living persons. In poetry the majority of people seek the passion
and pain of the human being behind the poet. Paintings attract them if they find in them, figures of men or women it would be
interesting to meet.

It thus appears that to the majority of people aesthetic pleasure means a state of mind that is essentially indistinguishable from
their ordinary behaviour. It differs merely in accidental qualities, being perhaps less utilitarian, more intense and free from painful
consequences. But the object toward which their attention and, consequently, all their other mental activities are directed is the
same as in daily life: people and passions. When forced to consider artistic forms proper for example, in some surrealistic or
abstract art most people will only tolerate them if they do not interfere with their perception of human forms and fates. As soon
as purely aesthetic elements predominate and the story of John and Susie grows elusive, most people feel out of their depth and
are at a loss as to what to make of the scene, the book, or the painting. A work of art vanishes from sight for a beholder who seeks
in that work of art nothing but the moving fate of John and Susie or Tristan and Isolde. Unaccustomed to behaving in any mode
except the practical one in which feelings are aroused and emotional involvement ensues, most people are unsure how to respond
to a work that does not invite sentimental intervention.

Now this is a point that has to be made perfectly clear. Neither grieving nor rejoicing at such human destinies as those presented
by a work of art begins to define true artistic pleasure; indeed, is preoccupation with the human content of the work in principle
incompatible with aesthetic enjoyment proper.
According to the passage, most people are not attracted towards modern art because
A)They consider modern art to be reserved for the elite few
B)They understand little of what is portrayed through these works, depending instead on the views of the critics for their
understanding
C)They are irked by the social messages implied by the modern art
D)They find in modern art, little of human interest to engage them
Explanation:-
The author states that the object towards ..passions. Hence if there is little which they can relate to in a particular thing
most people are not interested in that thing.

DIRECTIONS for the question : Read the passage and answer the question based on it.
Question No. : 95
If the new art is not accessible to everyone, which certainly seems to be the case, this implies that its impulses are not of a
generically human kind. It is an art not for people in general but for a special class who may not be better but who are evidently
different.

Before we go further, one point must be clarified. What is it that the majority of people call aesthetic pleasure? What happens in
their minds when they like a work of art; for example, a play? The answer is easy. They like a play when they become interested
in the human destinies that are represented, when the love and hatred, the joys and sorrows of the dramatic personages so move
them that they participate in it all as though it were happening in real life. And they call a work good if it succeeds in creating the
illusion necessary to make the imaginary personages appear like living persons. In poetry the majority of people seek the passion
and pain of the human being behind the poet. Paintings attract them if they find in them, figures of men or women it would be
interesting to meet.

It thus appears that to the majority of people aesthetic pleasure means a state of mind that is essentially indistinguishable from
their ordinary behaviour. It differs merely in accidental qualities, being perhaps less utilitarian, more intense and free from painful
http://www.hitbullseye.com/frmTestView.aspx

56/59

7/7/2015

about:blank

consequences. But the object toward which their attention and, consequently, all their other mental activities are directed is the
same as in daily life: people and passions. When forced to consider artistic forms proper for example, in some surrealistic or
abstract art most people will only tolerate them if they do not interfere with their perception of human forms and fates. As soon
as purely aesthetic elements predominate and the story of John and Susie grows elusive, most people feel out of their depth and
are at a loss as to what to make of the scene, the book, or the painting. A work of art vanishes from sight for a beholder who seeks
in that work of art nothing but the moving fate of John and Susie or Tristan and Isolde. Unaccustomed to behaving in any mode
except the practical one in which feelings are aroused and emotional involvement ensues, most people are unsure how to respond
to a work that does not invite sentimental intervention.

Now this is a point that has to be made perfectly clear. Neither grieving nor rejoicing at such human destinies as those presented
by a work of art begins to define true artistic pleasure; indeed, is preoccupation with the human content of the work in principle
incompatible with aesthetic enjoyment proper.
The authors assumption in the final paragraph is that
A)Aesthetic pleasure is a response to the purely artistic elements in a work of art
B)Aesthetic enjoyment of a work of art must focus on the artists intentions as much as on the artists actual accomplishments
C)The majority of people trying to interpret a work of art will concentrate on the artistic technique
D)The evocation of emotional responses by a traditional work of art depends on the moral conventions of the artists society
Explanation:-
The author is stating that any relation to human content in art is not compatible with aesthetic enjoyment. The author assumes
here that aesthetic pleasure is a behaviour stimulated by the artistic elements only and not by any human content visible to the
person.
DIRECTIONS for the question: Read the information given below and answer the question that follows.
Question No. : 96
K, L, M and N are four friends who live in the same area and work for the children in a nearby slum. One of them cooks the
childrens breakfast and each of the other three teach a different subject from Mathematics, English and Drawing. They study
Mechanical engineering, Psychology, Medicine and Economics. Neither the Economics nor the Psychology student cooks breakfast.
L does not teach English. N is the medical student and he does not cook breakfast. M teaches Drawing. K studies Mechnical
engineering. Who teaches Mathematics?
A)K

B)L

C)M

D)N

Explanation:-
Since the Economics, Psychology and Medicine students do not cook breakfast, the Mechanical Engineering student, K, cooks
breakfast. M teaches Drawing. Now since L does not teach English, she must be teaching Maths.

DIRECTIONS for the question: In the given paragraph, the last line has been deleted. Choose the option that logically follows the
paragraph.
Question No. : 97
Hindi Films and TV programs have often been blamed for making todays young men more violent. This however could be putting
the cart before the horse; it is possible that youth are attracted to films and such TV programs because the characters these
programs have, mirror their own images.
A)The fact is that action-oriented films possibly help as pressure-release valves for the youth today
B)
A university professor did some research recently with some of his male students, and concluded that there is a higher
probability that these students consider films such as Agneepath and Ghajni, as being highly realistic
C)In other words, an unusual interest in action programming may be an indication of a violence-prone personality rather

than an incitement to violence


D)Hence the censor board should ensure that films with a lot of explicit violence are asked to tone down these scenes
Explanation:-
The paragraph states that films and TV programs could be a reflection of what is present in the society rather than the other way
round. Option 3 is another way of saying the same. If the scenes are toned down, what they reflect may not be reality.
DIRECTIONS for the question: Read the information given below and answer the question that follows.
http://www.hitbullseye.com/frmTestView.aspx

57/59

7/7/2015

about:blank

Question No. : 98
In order to gain full course credit for her tour of a foreign city, Sue must visit exactly seven famous points of interest - a factory, a
garden, the harbour, a library, a museum, a palace and a theatre. Any tour plan that Sue devises will allow her to keep to her
timetable and is thus acceptable, except that she must plan her tour to conform to the following conditions.

The factory must be one of the first three points visited.


The harbour must be visited immediately before the garden.
The library can be neither the first nor the last point visited.
The museum must be either the first or the last point visited.
The palace must be one of the last three points visited.

Sue begins her tour with a visit to the harbour. Which of the following could be the fourth point of interest she would visit on the
tour?
A)the factory

B)the garden

C)the library

D)the museum

Explanation:-
From the given conditions: The factory must be at point 1 or 2 or 3. The harbour must not be point 7 and garden must not be
point 1. The library must be point 2 or 3 or 4 or 5 or 6. The museum must be point 1 or 7. The palace must be the point 5 or 6 or 7.
The harbour must be the point visited just before the garden.
If Sue begins her tour at the harbour then the garden must be point 2. So, the factory must be point 3 and the museum must be
point 7. The palace must be point 5 or 6. Library or theatre could be the 4th point.

DIRECTIONS for the question: Read the information given below and answer the question that follows.
Question No. : 99
In order to gain full course credit for her tour of a foreign city, Sue must visit exactly seven famous points of interest - a factory, a
garden, the harbour, a library, a museum, a palace and a theatre. Any tour plan that Sue devises will allow her to keep to her
timetable and is thus acceptable, except that she must plan her tour to conform to the following conditions.

The factory must be one of the first three points visited.


The harbour must be visited immediately before the garden.
The library can be neither the first nor the last point visited.
The museum must be either the first or the last point visited.
The palace must be one of the last three points visited.

If Sue visits exactly one point of interest between her visits to the factory and the palace, then that point must be either the

A)garden or the harbour

B)Library or the theatre

C)harbour or the museum

D)Library or the museum

Explanation:-
From the given conditions: The factory must be at point 1 or 2 or 3. The harbour must not be point 7 and garden must not be
point 1. The library must be point 2 or 3 or 4 or 5 or 6. The museum must be point 1 or 7. The palace must be the point 5 or 6 or 7.
The harbour must be the point visited just before the garden.
1 2 3 4 5 6 7 F PSince the harbour must be immediately followed by the garden, so neither the
harbour nor the garden will be that point between the factory and the palace. So, options (A) and (C) are eliminated. Since
museum must be either point 1 or 7, so option (D) is also eliminated. Hence option (B) is the answer.

DIRECTIONS for the question: In the sentence given below one word has been used in different ways. Select the one in which the
usage is incorrect.
Question No. : 100
STAND
A)The stars in the flag stand for the thirteen original colonies B)You must act respectful to him; he stands upon ceremony
C)Why dont you stand up to your boss when you know youre right?
D)That machine wont stand. You should get a dependable kind

http://www.hitbullseye.com/frmTestView.aspx

58/59

7/7/2015

about:blank

D)That machine wont stand. You should get a dependable kind


Explanation:-
Should have read stand up to last. 2 - to stand upon be strict about observing formality, ones rights etc. 3 - to stand up
to oppose.

http://www.hitbullseye.com/frmTestView.aspx

59/59

You might also like